MPRE Practice Qs

Pataasin ang iyong marka sa homework at exams ngayon gamit ang Quizwiz!

A sales manager testified before a federal grand jury that was investigating price-fixing in the automobile tire industry. Ultimately, the grand jury indicted the sales manager for price-fixing, a felony under the Sherman Act. After his indictment, the sales manager sought to hire a prominent attorney to represent him at his criminal trial. The sales manager is a middle class business executive with enough savings to pay for private counsel. He told his attorney in confidence that he had lied to the grand jury about several meetings he had had with competitors. Furthermore, he told her that he wanted to plead not guilty to the criminal charge and that he intended to testify at trial as he did before the grand jury. Which of the following would be proper for the attorney to do in this situation?

A - Decline to represent the sales manager. B - Agree to represent the sales manager and tell no one what he told her. C - Inform the sales manager that unless he pleads guilty to the criminal charge, she will tell the prosecutor about his false testimony before the grand jury. Incorrect D - Decline to represent the sales manager and inform the prosecutor about his false testimony before the grand jury. The attorney has no duty to represent the sales manager, so (A) is proper. (B) is improper because the attorney may only represent the sales manager if he does not insist on testifying falsely. [See ABA Model Rule 3.3(a)(3)] (C) is improper because it is a form of extortion. (D) is improper because the sales manager's confession to past perjury is protected by the duty of confidentiality.

An attorney is representing a defendant on trial for armed robbery of a liquor store. The defendant tells the attorney in confidence that at the time in question, he was sitting at home watching television with his aged mother, and that his mother can confirm his alibi. The attorney interviews the mother, who solemnly confirms the defendant's story. After talking with her, the attorney strongly suspects that she is lying to protect the defendant. The attorney does not know for sure that the defendant and his mother are lying, but every instinct tells him that they are. The attorney has warned both of them about the dangers of perjury, but both have insisted that they want to testify to the alibi at trial. May the attorney call the defendant, or his mother, or both, as trial witnesses?

AYes, as to both the defendant and his mother. need actual knowledge of false

A police officer was charged with murder. He is alleged to have savagely beaten and ultimately killed a teenage gang member in the course of an arrest. Neither the police department nor the officer's union was willing to provide legal counsel for his defense, and the officer himself lacked funds to hire private counsel. The public defender's office could not represent him due to a conflict of interest from a related case. The trial court therefore appointed a lawyer to defend the officer. The lawyer is only three years out of law school. The lawyer practices criminal defense, but he has never handled a murder case before. For which of the following reasons may the lawyer decline the court appointment? Based on what he has read in the newspapers, he sincerely believes that the officer is guilty. He has no experience in the defense of a murder case. He is of the same race as the teenage victim, and he is in sympathy with the plight of young gang members. He recently was diagnosed with severe depression, which is affecting his ability to handle his existing caseload.

A lawyer is subject to discipline for trying to avoid a court appointment without good cause. [ABA Model Rule 6.2] (D) is good cause because the lawyer's severe depression is preventing him from handling his existing caseload competently. [See comment 2 to ABA Model Rule 6.2-good cause exists if the lawyer could not handle the matter competently; and see ABA Model Rule 1.16(a)(2)-requiring withdrawal when the lawyer's physical or mental condition materially impairs the lawyer's ability to represent the client] (A) is not good cause because a lawyer's belief that the defendant is guilty is not a sufficient reason to turn down a court appointment. Competent defense of a murder case certainly does not require a defense lawyer to believe in the client's innocence. (B) is not good cause because the facts state that the lawyer is three years out of law school and practices criminal defense law. That indicates that his training in criminal law and procedure is recent, and that he knows how to defend a criminal case, even though he has not handled a murder case before. Thus, the lawyer cannot claim lack of competence as an excuse for turning down the appointment. [See Hazard & Hodes, §51.3] ABA Model Rule 6.2(c) recognizes that a lawyer may turn down an appointment if the client or cause is so repugnant to him as to interfere in the lawyer-client relationship. Neither the lawyer's race nor his sympathy for young people who get involved with gangs should be regarded, without more, as likely to interfere with the lawyer's ability to represent the police officer competently. Thus, (C) is not good cause.

In the most recent election, a lawyer who practices election law represented a candidate who, several days before the election, told the lawyer in confidence that he had hired some gangs of thugs to frighten voters away from the polls in neighborhoods where most people would vote for his opponent. The lawyer was shocked and immediately advised the candidate to call off the thugs or withdraw from the race. The candidate refused to do either, whereupon the lawyer withdrew as the candidate's lawyer. The lawyer did not, however, tell anyone about the candidate's evil plan. On election day, the candidate's thugs did what the candidate paid them to do, and the candidate defeated his opponent by a narrow margin. A few days later, the opponent learned what the thugs had done. The opponent was furious and called a press conference at which he accused the candidate and the lawyer of conspiring to intimidate his supporters and keep them away from the polls. The local newspaper printed his allegations in a front-page story, accompanied by a large photograph of the candidate and the lawyer smiling at each other. Which of the following propositions is false?

After the opponent's press conference and the newspaper story, but before any kind of formal proceeding, it would be improper for the lawyer to disclose what the candidate told him in confidence about the thug plan and about his own response. 1.6(b)(5) allows a lawyer to reveal confidential information "to establish a defense to a criminal charge or civil claim against the lawyer based upon conduct in which the client was involved, or to respond to allegations in any proceeding concerning the lawyer's representation of the client." The lawyer's right to disclose in self-protection does not require that a criminal charge, civil complaint, disciplinary case, or other formal proceeding has already started. "The lawyer's right to respond arises when an assertion of complicity has been made." [Comment 10 to ABA Model Rule 1.6] The opponent's press conference and the newspaper story amply satisfy that requirement.

A paralegal works for the law firm of Alpha & Beta. Her direct supervisor is partner Alpha, whose practice is limited to international trade law. Partner Beta is the firm's leading trial lawyer, both in commercial and personal injury cases. On her way to work one morning, the paralegal saw a pedestrian run down in a crosswalk by a speeding car. The paralegal rendered first aid, and while she was waiting with the pedestrian for the ambulance, the paralegal provided the pedestrian with a business card and urged him to call the firm to obtain legal representation in connection with his injuries. When she got to work, she told partner Alpha what she had done. Alpha admonished the paralegal not to hand out the firm's cards in such situations, but he did not discuss the matter with partner Beta. Is Alpha subject to discipline?

Alpha is subject to discipline for failing to warn Beta not to take the case. If the paralegal were a lawyer, her conduct would violate ABA Model Rule 7.3(b), which prohibits in-person solicitation. The partners in a firm are responsible for educating their nonlawyer employees about ethics issues and making reasonable efforts to assure that those employees comply with ethics rules. [ABA Model Rule 5.3(a)] Moreover, a partner is subject to discipline if he learns about the violation of an ethics rule by a nonlawyer employee "when its consequences can be avoided or mitigated," but the partner "fails to take reasonable remedial action." In this case, the consequences of the paralegal's solicitation could have been avoided by warning Beta not to take the pedestrian's case. Because he failed to warn Beta, Alpha is subject to discipline. (B) is wrong because it is too broad. A lawyer's responsibility for a nonlawyer employee's ethics violation is limited to situations in which the lawyer orders it, ratifies it, or learns about it in time to remedy it and does not do so. [ABA Model Rule 5.3(c)] (C) is wrong because even though people are generally free to recommend a lawyer to someone else, that does not allow the paralegal to solicit business for the firm that employs her. [ABA Model Rule 8.4(a)] (D) is wrong because Alpha and the other partners in the firm had a duty to educate the paralegal about ethics rules. [ABA Model Rule 5.3(a)] Furthermore, even if the paralegal acted innocently, that does not excuse Alpha's failure to warn Beta not to take the case.

A judge is known to grant continuances whenever requested by an attorney, regardless of the substantiality of the attorney's grounds. He has turned down a continuance request on occasion, but such occasions are so few and far between that local attorneys are shocked when they hear of them. When queried by a judicial colleague about his policy on continuances, the judge told her that granting continuances gives the parties that much more time to come to their senses and settle, which saves both the parties and the taxpayers the expense of a full-blown trial. Is the judge's policy of granting continuances to promote settlements proper?

No, because judges have a duty to expedite litigation. CJC Rule 2.5 requires a judge to perform his duties competently and diligently. Comment 3 to that rule explains that a judge should ensure that court officials, litigants, and their lawyers cooperate with the judge in disposing of matters promptly. A judge may choose to promote settlements only if such policy does not conflict with other requirements for proper judicial conduct.

A new associate at a law firm was asked to help a partner advise a state university on how to comply with a federal statute that requires colleges and universities to make many changes in their facilities to accommodate students with disabilities. After graduating from law school, the associate had worked on the congressional staff of a United States senator. In that role, she personally drafted a bill that was ultimately enacted as the federal statute. In light of the associate's earlier role as the drafter of the federal statute, which of the two lawyers may work on the matter?

Both the partner and the associate may work on the matter, assuming the associate complies with the applicable federal statutes and regulations concerning former government employees. Drafting a piece of legislation is not regarded as a "matter" for purposes of the legal ethics rules on former government employees. [ABA Model Rule 1.11(e); ABA Formal Op. 342 (1975)] Therefore, the associate may advise the state university. Because the associate is not disqualified, neither is her firm. Thus, the partner may also work on this project. (A) is wrong because, as discussed above, the associate's congressional work disqualifies neither the associate nor the partner. (B) is wrong because it states one of the requirements for the partner's representation had the associate been disqualified. As discussed above, the associate is not disqualified and thus need not be screened off. (D) is wrong for the reasons stated above. Moreover, it is not the state university that would need protection if this were a "matter" for the purpose of disqualification under the conflict of interest rules. In that case, the associate would have been screened off, the associate would not be apportioned any part of the fee, and written notice would be promptly given to the government agency.

An attorney knows that the statute of limitations on her client's claim has run. However, the statute of limitations is an affirmative defense that the defendant in the case would waive if she failed to plead it. The attorney's client is willing to incur the legal fees and court costs of filing the lawsuit, and understands the risks. What may the attorney do?

File the suit because her client is willing to incur the legal fees and court costs. The statute of limitations merely destroys the remedy and not the right. Unless the defendant pleads the statute of limitations, the claim is valid. no duty to inform ct that SOL has run

A lawyer represents a plaintiff in a sexual harassment case against her employer. The employer is represented by its regular corporate counsel. The plaintiff, who works on an assembly line, alleges that she was repeatedly harassed by the foreman on her work shift. Furthermore, she alleges that the plant manager was aware of this misconduct and did nothing to stop it. The plaintiff tells her attorney that two of her co-workers on the assembly line witnessed harassment incidents, but neither reported the incidents to supervisory personnel. One of the witnesses quit working for the employer at about the time the plaintiff filed her lawsuit. The lawyer wants to do some fact investigation before he starts discovery in the case, and wishes to interview the plant manager, the foreman, and the two co-worker witnesses. Which of the following best states proper conduct for the lawyer in interviewing these individuals?

He may freely interview both witnesses, but he must obtain defense counsel's consent to interview the foreman and the plant manager. A lawyer must obtain the consent of an organization's counsel before communicating with: (i) a person who supervises, directs, or consults with the organization's lawyers about the matter (here, the foreman and the plant manager); (ii) a person whose conduct may be imputed to the organization for purposes of criminal or civil liability (the foreman and the plant manager again); or (iii) a person whose statements may constitute an admission by the organization (again, the foreman and the plant manager). [Comment 7 to ABA Model Rule 4.2] Consent is not needed, however, before talking with a former employee. Thus, the lawyer can speak to the witness who no longer works for the defendant without defense counsel's consent. Likewise, consent of defense counsel is not necessary to interview an employee who is merely a witness to the incident in question and is neither a management employee nor involved in the incident. Thus, the lawyer may interview the witness who still works for the defendant without defense counsel's consent.

A judge serves on a state trial court that has nine other judges. Her husband is a life insurance salesman for a large life insurance company. The life insurance company is occasionally a litigant in the court on which the judge sits. Every year the life insurance company runs a national sales contest in which the person who sells the most life insurance during the year receives a valuable prize. The judge's husband won this year and took the judge on an all-expense-paid vacation in Europe. She did not make a public report of the prize. Was it proper for the judge to allow her husband to accept the prize and take her on the European vacation?

It was proper for the judge to allow her husband to accept the prize because acceptance thereof does not reasonably appear to undermine the judge's integrity or impartiality. A judge may accept benefits associated with her spouse's business activity that incidentally benefit the judge. [CJC Rule 3.13(B)(8)] (B) is wrong because it ignores the general rule on family members' accepting gifts and other benefits. (C) is wrong because such benefits of a spouse are not subject to the public reporting requirement. [CJC Rule 3.13(B)] (D) is wrong because if the prize is proper under CJC Rule 3.13(B)(8), it does not become improper simply because the insurance company may later appear as a litigant in the judge's court.

A client lives in State A and is a regular client of an attorney who is admitted to practice only in State A. When the client was on vacation in distant State B, she was injured in a car accident caused by a resident of State B. The client hired the attorney to represent her in a civil action against the State B driver. For reasons of jurisdiction and venue, the case had to be filed and tried in State B. The written fee agreement between the client and the attorney provided that: (1) The attorney would assume full responsibility for the case as lead lawyer; (2) The client would pay the attorney 40% of the net recovery after deduction of litigation expenses; (3) The attorney would associate a State B lawyer to serve as trial counsel in State B; (4) The State B attorney would assume responsibility only for his work as trial counsel; and (5) The attorney would pay the State B attorney an appropriate portion of the 40% contingent fee. Would it be proper for the attorney to split his fee with the State B attorney under the circumstances described above?

It would not be proper for the attorney to split his fee with the State B attorney because the written fee agreement with the client does not comply with the ABA Model Rules. ABA Model Rule 1.5(e) allows a lawyer to split a fee with a lawyer who is not in his firm if: (i) the total fee is reasonable; (ii) the split is in proportion to the services rendered by each lawyer, or in some other proportion if each lawyer assumes joint responsibility for the matter; and (iii) the client agrees to the split in a writing that discloses the share that each lawyer will receive. Here, the written fee agreement did not specify the share that each lawyer will receive; thus, a fee split between the attorney and the State B attorney would be improper. (A) is wrong because there is no requirement that a lawyer be licensed in the same state as the attorney with whom he is splitting a fee. (C) is wrong because the written fee agreement with the client did not indicate the share that each lawyer will receive, and thus the agreement was improper regardless of whether the State B attorney was assuming responsibility for his work. (D) is wrong because even though there was a fee agreement, it did not comply with the ABA Model Rules.

A solo practitioner is one of only three lawyers in a small town. The solo practitioner is presently defending a client in a criminal action for assault and battery. This morning one of the solo practitioner's regular clients, a gas and grocery store, asked the solo practitioner to sue the same client to recover a past due amount on a gasoline and grocery charge account. Would it be proper for the solo practitioner to represent the gas and grocery store in the charge account case?

It would not be proper for the solo practitioner to represent the grocery store unless both the client and the grocery store give informed consent, confirmed in writing, to the representation. A lawyer who is presently representing a client in one litigation matter should not simultaneously oppose that client in a different litigation matter, even if the two matters are unrelated. [ABA Model Rule 1.7(a) and comment 6] The purpose of the rule is to avoid putting the client into the difficult position of treating the lawyer simultaneously as friend and foe. The conflict can be solved only by informed consent, confirmed in writing, from both the client and the grocery store. (A) is wrong because the rule prohibiting the representation applies regardless of whether the solo practitioner has obtained relevant confidential information. (B) is wrong because the conflict is not obviated by the shortage of legal talent in the small town. (C) is wrong because the Rule applies even if the two cases are unrelated.

An attorney represents a plaintiff in a personal injury suit arising out of a tour bus accident in Hawaii. Nearly all of the eyewitnesses were tourists who have now returned home to the mainland. Without notifying the defense attorney, the plaintiff's attorney has interviewed most of the witnesses by phone. By far the most compelling witness, and the one most favorable to the plaintiff, is a librarian who lives in North Dakota and had spent the bulk of her life savings on a vacation to Hawaii. It was on this vacation that she witnessed the tour bus accident. She is a very appealing witness, and the attorney is confident that if a jury saw her testify personally, the plaintiff would win his suit. The attorney tells the witness that if she is willing to come to Hawaii for one week to testify, he will pay for her plane tickets, an oceanfront hotel room at a first class hotel, all meals, and one week's salary for her lost time. This is the same offer that the attorney makes to all witnesses traveling to testify in any of his cases. The witness, who cannot believe her good fortune, readily agrees. Is the attorney subject to discipline?

No, because a lawyer may pay a witness's reasonable expenses and lost wages. including travel expenses, hotel, and meals. [Comment 3 to ABA Model Rule 3.4; Restatement §117, comment b]

The attorney general's office does not include any lawyers who are skilled in the field of condemnation law (the law of eminent domain). Consequently, whenever the state wants to use its power of eminent domain to condemn some private property for a public use, the attorney general must hire a private law firm to represent the state in the condemnation proceedings. In contrast to the paltry fees that the state pays to appointed defense counsel in criminal cases, the attorney general pays quite handsomely for condemnation work. The attorney general is a partisan political position that is filled by a contested election every four years. A large state law firm limits its practice to condemnation law. The founding partner is an 87-year-old multimillionaire who remains active on the firm's management committee. When it is time to elect a new attorney general, the partner makes large donations from his personal wealth to each candidate who has any reasonable chance of becoming the next attorney general. The other members of the firm's management committee know about the partner's contributions, and they have formally and informally expressed the firm's thanks for helping the firm obtain future appointments by the attorney general. May the firm accept an appointment from the new attorney general to represent the state in a condemnation case?

No, because a lawyer or law firm must not accept appointed legal work from a governmental official after making a political contribution for the purpose of obtaining such work. ABA Model Rule 7.6 prohibits "a lawyer or law firm" from accepting an appointed legal engagement if "the lawyer or law firm" makes a political contribution "for the purpose of obtaining or being considered for" that kind of legal engagement. The tricky part of this question is whether a political contribution by one of the firm's lawyers ought to bar the entire firm from taking subsequent appointments. Neither the Rule nor its comments speak directly to that point, but the purpose of the Rule would be served by imputing one lawyer's contribution to the entire firm, just as a conflict of interest would be imputed under ABA Model Rule 1.10(a). To adopt the opposite position would make ABA Model Rule 7.6 too easy to evade-the firm could simply ask its lawyers to make "pay-to-play" contributions from their own pockets. Using a different theory, at least the founding partner's colleagues on the management committee should be barred from accepting appointments from the attorney general because they knew about the partner's political contributions and thanked him rather than stopped him. [See ABA Model Rule 5.1(c)-ratification or acquiescence by managing lawyers] (B) is wrong for the reasons stated above. (A) is wrong because the partner's practice of contributing generously to all candidates who have a reasonable chance to win the attorney generalship demonstrates that his purpose is to secure business for the firm, not to participate legitimately in the political process. (D) is not as good as (C) because (D) relies on the outdated "appearance of impropriety" rubric of the old ABA Model Code, Canon 9. That rubric was cast aside in the ABA Model Rules-seeking to avoid even the "appearance of impropriety" is useful in a person's own moral creed, but it is too amorphous to be useful in a professional code of conduct.

A judge who was considered a great personal injury litigator when he was in private practice is trying a very complicated commercial law case. He has carefully listened to the opposing attorneys' arguments and has read the briefs several times. He has found neither the oral arguments nor the briefs to be very enlightening. A former law partner of the judge is considered to be one of the leading experts on commercial law in the state, and the judge wishes to lend the briefs to her, and have her write an advisory memorandum on the issues of the case. The judge sincerely feels that this will enable him to render a proper judgment in a difficult case, so he approaches his former law partner and obtains her help. Was it proper for the judge to seek such help from his former law partner?

No, because he did not give notice to the parties and allow them time to respond to the former law partner's memorandum.

The owner of a parcel of real estate on the east side of town wants to trade his land for a parcel of land owned by a woman on the west side of town. The owner of the west side property retains an attorney and tells him that she wishes to complete the deal as cheaply as possible. She asks the attorney to draw up papers for the transfer of property and asks him to order a title search and survey of the other party's property. The attorney recommended to her that a similar search and survey be conducted for her own property as well, but she replied that she wished to save money and did not need a search and survey for her property. The two parties exchange warranty deeds, and each takes possession of the other's property as agreed. A year later, the now-owner of the west side property-the property that did not have a title search and survey after the then-owner rejected the idea-contracts to sell the property. The prospective purchaser orders a title search and survey and discovers that there are defects that will substantially reduce the value of the property. The owner of this property eventually sells it at a price much lower than he could have commanded had the defects not been present. He sues the former owner for damages and receives an award of $10,000. The former owner, on the other hand, feels that her attorney is responsible for this, and she sues him to recover the $10,000. Is the attorney subject to liability for malpractice?

No, because he was following his client's instructions. NOT: No, because the buyer did not insist on the seller's presenting a title search and survey. - IRRELEVANT The attorney suggested a title search for the property. Upon learning that his client did not want to spend money for it, he was not required to order the search and pay for it himself, or to try to force her to pay for it. All he can do is give advice. If his client unwisely decides to accept only part of that advice, and suffers because of it, the client cannot successfully sue him for malpractice. The client's injury cannot be said to have been proximately caused by any negligence on the part of the lawyer.

The mayor of a small city is also a licensed attorney who has a law partnership with a fellow attorney in the city. Under the city's charter, the mayor has the authority to determine what issues are to be placed upon the agenda of the city council. Several council members have told the mayor that they would like to see a particular zoning measure placed upon the agenda. This proposed ordinance would ban commercial development of a certain area within the city limits. The mayor's law partner has been retained as attorney for a development company that has acquired land in the proposed noncommercial zoning area and has plans to construct a large shopping center there. The mayor has agreed to take no direct role in the representation, not to share any fees from the case, and not to attend any city council meetings at which the matter will be discussed. May the mayor's law partner represent the development company in this matter?

No, because of the mayor's position as mayor. If one lawyer within a firm has a conflict of interest and cannot take on a matter, no other lawyer in the firm may take on the matter either. [ABA Model Rule 1.10(a)] One situation that would create a concurrent conflict would be if there is a significant risk that the representation of a client will be materially limited by the lawyer's own interest or his responsibilities to another client, a former client, or a third person. [ABA Model Rule 1.7(a)(2)] The mayor would be prohibited from representing the development company in this matter because such representation would be materially limited by his responsibilities, as mayor, to the city. Thus, his law partner is also prohibited from such representation.

A man walked into a lawyer's office and explained that he is a dealer of illegal drugs, that one of his sales associates has just been arrested, and that he would like the lawyer to defend the sales associate. The drug dealer offered to leave a $10,000 retainer comprised of a shoebox of $100 bills. The lawyer was uncertain about getting involved with the drug dealer and especially about accepting a large sum in cash from the dealer. The lawyer told the dealer that he would think about his request and would let him know later that day. When the drug dealer left the office, the lawyer telephoned his friend and mentor, a retired judge. The lawyer told the judge the entire story, including the dealer's name, and asked the judge whether it would be ethical to defend the sales associate and accept the dealer's cash. Is the lawyer subject to discipline for telling the judge the whole story?

No, because the ethical duty of confidentiality has an exception that allows a lawyer to reveal confidential information to obtain legal ethics advice ABA Model Rule 1.6(b)(4) and comment 9 explain that a lawyer may reveal information that would otherwise be confidential if the lawyer's purpose is to obtain legal advice about complying with the legal ethics rules. (A) is wrong because the applicable doctrine here is the ethical duty of confidentiality, not the attorney-client privilege, and the ethical duty contains the exception described above. (B) is wrong because the exception to the ethical duty would apply in this situation, whether or not the lawyer identified the dealer by name. As a practical matter, however, a lawyer who discloses confidential information to obtain legal ethics advice may wish to couch the information hypothetically, in order to minimize the chance of harm to the client. [See comment 4 to ABA Model Rule 1.6] (C) is wrong because the exception to the ethical duty would apply in this situation whether the drug dealer or the sales associate is regarded as the client.

While she was attending law school at night, a law student served as a clerk for a judge of the county court. During the course of her employment, a public interest case came before the judge. The case was very complicated, and the law student did a lot of research on the case for the judge, submitting a number of memoranda on issues in the case. The judge always carefully supervised his clerks, and this case was no exception. He was exceptionally pleased with the care and high quality of the law student's work. Shortly after the judge handed down his final judgment, the law student graduated, was admitted to the bar, and accepted employment with a prominent local law firm. A few days later, the defendant from that case, who lost, appeared at the offices of the law firm. He told the interviewing attorney that his trial attorney was terrible. He wants the firm to handle his appeal. If the law firm accepts the client, and the former law student's supervising attorney asks her to handle the appeal, would it be proper for the former law student to do so?

No, because the former law student was previously personally and substantially involved in work on the case. (Note that ABA Model Rule 1.12(a) would permit the representation if both parties gave informed consent, confirmed in writing, to the representation, but there is no mention of both parties giving informed consent in these facts.)

An insurance company offers a legal services insurance policy. In return for a yearly premium, an insured will be reimbursed by the insurance company for a specified amount for legal services during the year. The insured selects a lawyer from a list of "authorized providers" supplied by the insurance company. Any lawyer who agrees to follow a maximum fee schedule set by the insurance company can become an "authorized provider." The insurance company solicits insurance sales by in-person and live telephone contact with potential insurance buyers, working systematically through local telephone directories. Will an attorney be subject to discipline if he becomes an "authorized provider" and receives clients through the insurance company's insurance plan?

No, because the insurance company does not specifically target persons whom it knows are in need of legal services in a particular matter covered by its insurance plan. The attorney will not be subject to discipline if he becomes an authorized provider under the insurance company's plan. The insurance company has set up a prepaid legal services plan of the kind referred to in the ABA Model Rules. A lawyer may receive legal business through such a plan, unless the operator of the plan uses live person-to-person contact to solicit people who it knows are in need of legal services in a particular matter covered by the plan. [ABA Model Rule 7.3(e)] (B) is wrong because there is no rule concerning the selection of counsel in a legal services insurance plan. (C) is also wrong because it would be an antitrust violation (and also an ethics violation) for a group of lawyers to conspire to follow a minimum or maximum fee schedule [see Arizona v. Maricopa County Medical Society, 457 U.S. 332 (1982)], but it is neither illegal nor unethical for a lawyer to agree to follow an insurance company's maximum fee schedule for work done for that company's insured. (D) is wrong because it is too broad. ABA Model Rule 7.3(e) permits a lawyer to participate in a legal services plan that uses live person-to-person contact, as long as it does not contact persons who are known to need legal services in a particular matter covered by the plan.

A district court judge heard through the "courthouse grapevine" that the district attorney was investigating corrupt practices in the courts and that the investigation focused on some as yet unascertained time in the past. The "rumor mill" also indicated that several judges and former judges were likely to be indicted for taking bribes to "fix" cases and to generate business for certain attorneys. The judge was alarmed at this news, and he telephoned a retired attorney, arranging to meet him for cocktails and dinner. At the restaurant, the judge slipped the maitre d' a 10-dollar bill to secure seating at a secluded corner booth. Over dinner, the judge told the retired attorney that he had accepted bribes in the past and that he did not know what he should do in light of the district attorney's investigation. The retired attorney advised the judge to do nothing. The judge picked up the $120 dinner tab, and the retired attorney thanked him for the fine meal. A month later, indictments were handed down against two sitting judges and three former judges. The judge was not among them, and it turned out that the period covered by the district attorney's investigation was prior to the judge's election to the bench. Six months after the indictments were announced, a member of the state appellate court died, and the governor announced that he was appointing the judge to serve the remaining three years of the justice's unexpired term. Must the retired attorney report his knowledge of corruption in office by the judge?

No, because the judge's disclosures to the retired attorney dealt with past crimes. Although the retired attorney has retired from active practice, it is clear from the facts that the judge consulted him as an attorney, rather than as a friend; thus, the retired attorney may not reveal the judge's disclosures of past crimes.

A consumer who bought a defective product that injured him hired a lawyer to represent him in a personal injury action against the large corporation that made the product. As the consumer and the lawyer discussed the case, the consumer stated that he probably would not agree to a settlement under $500,000. The lawyer agreed that the claim was worth at least that, but felt they would receive a much higher award if the case went to a jury. Shortly before the trial started, the lawyer for the corporation contacted the consumer's lawyer with a settlement offer of $150,000. The consumer's lawyer tried to call his client, but could not reach him. After two hours of trying to reach his client, the attorney called opposing counsel and rejected the offer. At trial, the jury awarded the consumer $1 million. Is the consumer's lawyer subject to malpractice liability for his actions?

No, because the jury award was much greater than the settlement offer.

Before a judge was elected to the bench, she and her law partner purchased a piece of property to be held in co-tenancy by the judge and her law partner. After the judge was elected to the bench, she agreed to pay her law partner an annual fee to manage the property because her own time would be severely limited by her judicial duties. The judge and her law partner meet every three months to discuss the status of the property. The law partner sometimes appears as an attorney in the judge's courtroom. Was it proper for the judge to make this arrangement with her law partner?

No, because the law partner appears in cases before the judge's court. A judge should refrain from financial and business dealings that involve her in frequent transactions with lawyers or persons likely to come before the court on which she sits. [CJC Rule 3.11(C)(3)] It makes no difference that the judge acquired the property before she became a judge.

For many years, a lawyer has represented a wealthy building contractor in a wide variety of legal matters, including disputes with suppliers and customers. The contractor is a contentious man; indeed, he is the lawyer's most lucrative client. The contractor built a single-family home for a married couple. When the contractor finished, the couple confronted him with a list of 289 items that they claimed were either construction defects or uncompleted tasks. The couple refused to pay the contractor the final 25% installment under their construction contract until all 289 items were remedied. The contractor contested all 289 items and demanded immediate payment. The couple refused to pay and refused to talk further with the contractor. At that point, the contractor called in the lawyer, who met with the couple and offered to represent both them and the contractor in trying to find an amicable solution to their dispute. The lawyer said that if his efforts failed, he would charge no fee; if his efforts produced an amicable solution, then his total fee would be $2,500, half to be paid by the couple and half by the contractor. May the lawyer represent both the couple and the contractor on those terms?

No, because the lawyer cannot reasonably believe that he can represent the couple competently and diligently in light of his relationship with the contractor. Here, the lawyer cannot reasonably believe that he can represent the couple competently and diligently in negotiating with the contractor, his longtime and most lucrative client. Moreover, the interests of the contractor and the couple are so "fundamentally antagonistic" that the conflict is "unconsentable."

Two attorneys, a brother and sister, were licensed to practice in a province in Canada. The sister moved to the United States and passed a state bar examination. She opened a law office in that state and had letterhead stationery printed which contained the names of both the sister and brother, followed by the words "Partners, Attorneys at Law." Is it proper for the sister to use such letterhead?

No, because the letterhead indicates that the sister is aiding the unauthorized practice of law. A lawyer who is licensed to practice in one state (or in a foreign country or province) is not, without more, entitled to practice in any other state. A lawyer is subject to discipline for practicing in a jurisdiction without being licensed to do so. Because there is no indication on the letterhead regarding limitation of the brother's practice to a province in Canada, the sister is abetting the unauthorized practice of law. [See ABA Model Rule 5.5(a)]

When a farmer sold the family farm back in 1967, he told his attorney that he wanted to reserve all of the subsurface rights-i.e., the rights to mine or drill for oil, gas, minerals, geothermal energy, and the like. The attorney drafted a deed that was supposed to accomplish that goal. Ten years later, the farmer died and the subsurface rights passed by his will to his beloved granddaughter. Thirty years later, the granddaughter received a letter from a petroleum exploration and drilling company offering to lease the subsurface drilling rights from her in return for $10,000 plus 15% of the wellhead selling price of all gas and oil obtained from the property. All the granddaughter had to do was sign the lease and supply the company with a letter from a qualified oil and gas lawyer, certifying that the granddaughter was indeed the owner of the subsurface rights. The granddaughter hired an oil and gas lawyer to research her title, write the letter, and send it to the company. After doing a little research, the oil and gas lawyer concluded that the farmer's attorney had bungled the deed. Instead of reserving the subsurface rights, the attorney had inadvertently sold them to the man who bought the farm. With a heavy heart, the oil and gas lawyer put her findings in an opinion letter and mailed duplicate originals to the company and the farmer's granddaughter. Was it proper to do so?

No, because the oil and gas lawyer should have obtained the granddaughter's informed consent before sending the letter to the company. The granddaughter hired the oil and gas lawyer to evaluate her title for the use of a third party, the petroleum exploration and drilling company. 2.3(b) states that when the lawyer knows (or should know) that the representation will harm the client, the lawyer should not provide the evaluation without first obtaining the client's informed consent.

A lawyer represented a corporation in some business negotiations with a construction company. Only four persons were present during the negotiations: (i) the corporation's president; (ii) the corporation's business lawyer; (iii) the construction company's chief executive officer ("CEO"); and (iv) the construction company's attorney. During the negotiations, the corporation's president and its business lawyer clearly heard the construction company's CEO make a certain representation that was vital to the success of the negotiation. Based on this representation, the parties reached an oral, handshake agreement to pursue a certain business opportunity as a joint venture. Six months later, after the corporation had invested $11 million in the joint venture, the corporation discovered that the representation made by the construction company's CEO was false and that he undoubtedly knew it was false when he made it. Due to the false representation, the joint venture failed, and the corporation lost its $11 million. The corporation sued the construction company and its CEO in federal court for intentional misrepresentation. The corporation selected one of its lawyer's law partners as its trial counsel. The defendants denied making the representation. The corporation's final pretrial statement listed the corporation's president and the lawyer as witnesses for the corporation, stating that they would testify that they heard the construction company's CEO make the representation. Counsel for the defendants then moved to disqualify the lawyer's law partner as trial counsel due to the lawyer's acting as a witness for the corporation. Is the law partner subject to disqualification?

No, because there is no conflict of interest presented by the lawyer's role as a witness for the corporation, and because a lawyer is ordinarily allowed to serve as trial counsel in a case where his law partner will testify on behalf of his client. NOT: No, because the lawyer's testimony will be merely corroborative; he will simply confirm the corporation president's testimony that the construction company's CEO made the representation at the meeting. The rule that conflicts of interest are imputed to all lawyers who work in the same office does not ordinarily apply to the advocate-witness rule. [See ABA Model Rule 3.7(b)] - merely corroborative isnt a thing The imputation rule does apply, however, if there is some other kind of conflict of interest caused by a lawyer's role as witness. [Id.] Suppose, for example, that the lawyer's testimony would contradict the corporation president's testimony rather than corroborate it. If that were true, it would create a conflict under ABA Model Rule 1.7(a) between the lawyer's own interest (telling the truth) and the corporation's interest (proving the representation was made). That conflict could well be so serious that even the corporation's informed consent could not overcome it.

The law of a particular state prohibits agreements not to compete, except for agreements that are ancillary to the sale of a business or professional practice and are reasonable in both duration and geographic scope. For 20 years, an attorney practiced patent, copyright, and trademark law in an area within the state. Seeking a new challenge, the attorney entered the political race for a trial court judgeship. He won a four-year term. Before taking the oath of judicial office, the attorney sold his entire law practice to a young lawyer for $150,000. In the sale contract, the attorney promised the younger lawyer not to re-enter the practice of patent, copyright, or trademark law in the area for five years. At the end of his four-year term, the attorney ran for re-election to his judgeship; to everyone's great surprise, he lost the election to a much less qualified opponent. Because he needed to earn a living, the attorney immediately re-entered the practice of patent, copyright, and trademark law in the area in which he formerly had practiced. Is the attorney subject to discipline?

No, but the attorney could be subject to civil liability to the younger lawyer in a suit to enforce the agreement not to compete, assuming that the agreement was reasonable in duration and geographic scope. ABA Model Rule 1.17 permits a practicing lawyer to sell his entire law practice or an area of his law practice to one or more lawyers or law firms. One of the conditions specified in ABA Model Rule 1.17 is that the seller must cease "to engage in the private practice of law . . . in the [jurisdiction or geographic area] in which the practice has been conducted." [ABA Model Rule 1.17(a)] The attorney's re-entry into private practice does not violate ABA Model Rule 1.17 because it was caused by an "unanticipated change in circumstances"-i.e., his failure to win re-election.

A lawyer practices in State A. State A's rules of legal ethics depart from the ABA Model Rules in one significant respect: State A has no "financial injury" exception to the lawyer's duty of confidentiality. Thus, when a State A lawyer learns in confidence that her client is about to use her legal services to inflict serious financial injury on someone, the lawyer may withdraw, but she must not reveal what she learned in confidence. The lawyer limits her practice to federal securities law, and she regularly appears before the Securities and Exchange Commission ("SEC"). One of her major clients is a company that makes and sells cotton textiles. The company's shares are traded on the New York Stock Exchange and in securities matters the company is regulated by the SEC. While working on an SEC registration statement for the company, the lawyer learned in confidence that three of the company's top executives were cooperating in a scheme to loot the company of millions of dollars. If their scheme continues, it could drive the company into insolvency. The lawyer alerted the chief legal officer of the company to the situation, but he did nothing. She then alerted the chief executive officer, who also did nothing. Finally, she alerted the six outside members of the board of directors, but they too failed to act. In disgust, the lawyer withdrew from the matter and vowed never again to represent the company. Must the lawyer now tell the SEC about the scheme?

No, the SEC's regulations give her discretion to either reveal or not reveal the matter to the SEC. The SEC's regulations under the Sarbanes-Oxley Act permit, but do not require, a securities lawyer to reveal a client's confidential information to the SEC when the lawyer reasonably thinks that doing so is necessary to prevent or rectify a securities act violation (or similar law violation) that is likely to cause substantial financial injury to the client or its shareholders.

A man walked into the law offices of a lawyer who, because she was not busy at the time, agreed to talk to the man right away. The man told the lawyer that he was concerned that he might be indicted soon. He explained the details of his predicament at length to the lawyer, but after he finished, the lawyer explained that she only handled civil matters and urged the man to consult with a friend who was a criminal lawyer. The man went on to retain the friend. A few days after her interview with the man, the lawyer read a news item announcing the man's indictment. It quoted the district attorney at some length. After reading the article, the lawyer became convinced that something the man had told her during their interview would probably exonerate him or, at the very least, lead to a reduction in the charges against him if the district attorney became aware of the information in her possession. May the atty reveal this info?

No, unless the man consents to the disclosure An attorney may reveal or use confidential information if the client gives informed consent. [See ABA Model Rules 1.6, 1.18] man consulted the lawyer in her capacity as an attorney, and it is irrelevant that he did not retain her attorney may not reveal information, even if it will help the client, unless the client consents or the information falls into recognized exceptions to the confidentiality rule, which are not present here.

Without the assistance of independent counsel, a client signed a retainer agreement with his lawyer that included the following provisions: (a) The lawyer would spend no more than 20 hours on the matter; (b) The client must arbitrate any legal malpractice claims; and (c) The lawyer's liability for any malpractice issues would be capped at $10,000. What (if any) parts of this agreement are valid?

Parts (a) and (b) are valid. A lawyer may limit the scope of her representation (as in (a)) and may agree prospectively with a client to arbitrate all matters (as in (b)). Part (c) is not valid. A lawyer cannot limit her liability for malpractice unless the client was independently represented in negotiating the agreement. PR0316B

Client A and Client B, both defendants in a tort case, want to hire Lawyer L to defend them jointly. After reviewing the facts of the case, Lawyer L determined that she could effectively represent both clients. She disclosed the potential conflict to Client A and Client B, and they both gave their informed consent in writing. During discovery, Lawyer L concludes that the best defense for Client A would be to focus the blame on Client B. She then determines that it would not be reasonable to advise Client A and Client B to continue to be jointly represented. What options are available to Lawyer L?

She can withdraw from representing both parties, or continue to represent one party as long as the other grants informed consent, confirmed in writing

An attorney who was popular in the legal community decided to run against an incumbent judge in the forthcoming election. The judge was widely regarded by members of the local bar as a "party hack," who had no business being on the bench. The opposition party was very pleased to be able to slate the attorney because she had a high reputation for intelligence, honesty, and overall competence as an attorney. The attorney realizes that she will have to fight an uphill battle to unseat the judge because her political party is a minority party in the county and most voters know very little about judges and candidates for the judiciary and therefore, voters are likely to vote a straight ticket for judges of their own political party. The attorney wants the public to know that the judge has been a poor judge, but she also wishes to comply with all ethical rules governing judicial campaigns. The attorney's best friend and chief advisor, also a lawyer, suggests that the attorney should make some prepared statements during her campaign. Assuming that all the facts cited in the statements are accurate, which statement would not be proper for the attorney to make during her judicial campaign? That her opponent has had the highest percentage of cases reversed on appeal of any judge in the state over the past two years. That a recent newspaper article comparing judges of the county stated that her opponent had handed out an average sentence of only two years to persons convicted of serious felonies, and that she would not be soft on crime. That 18 months ago, her opponent was publicly disciplined by the state's judicial conduct board. That a recent poll taken by the local bar association indicated that a majority of bar association members feel that her opponent lacks the proper judicial temperament.

That a recent newspaper article comparing judges of the county stated that her opponent had handed out an average sentence of only two years to persons convicted of serious felonies, and that she would not be soft on crime. Not: That her opponent has had the highest percentage of cases reversed on appeal of any judge in the state over the past two years. That 18 months ago, her opponent was publicly disciplined by the state's judicial conduct board. That a recent poll taken by the local bar association indicated that a majority of bar association members feel that her opponent lacks the proper judicial temperament. ABA Model Rule 8.2(a) bars attorneys from making false statements about candidates for the judiciary. Furthermore, CJC Rule 4.1(A)(11) prohibits a judicial candidate from knowingly, or with reckless disregard for the truth, making a false or misleading statement. CJC Rule 4.1(A)(13), which prohibits a judicial candidate from making pledges, promises, or commitments, with respect to cases, controversies, or issues that are likely to come before the court, that are inconsistent with the impartial performance of the adjudicative duties of the office.

A bank operates a professional referral hotline for its depositors. Any bank depositor who needs to find a physician, lawyer, accountant, dentist, or the like can telephone the hotline and obtain a free referral from lists of professionals compiled by the bank. The lists are limited to professionals who maintain an average balance of at least $10,000 in an account at the bank, but the professional does not pay a fee to the bank for receiving a particular referral. A lawyer keeps $10,000 on deposit with the bank for the express purpose of being included on its lawyer referral list. Is this arrangement proper?

The arrangement is not proper because the lawyer is required to keep $10,000 on deposit to be included on the list. A lawyer may not give "anything of value" to a person for recommending the lawyer's services. [ABA Model Rule 7.2(b)] The bank benefits in many ways by increasing the amount of its deposits; for example, its deposits determine how much it can lend to borrowers. Thus, obtaining deposits from lawyers is of value to the bank, and that is one reason it has devised the referral scheme. (A) is wrong because although a lawyer may pay the usual charges of a not-for-profit or qualified lawyer referral service [ABA Model Rule 7.2(b)], banks operate for profit, and there is no indication that the bank has been approved by the appropriate regulatory authority as a qualified lawyer referral service. (B) is wrong because the lawyer is giving something of value for the referrals, as explained above, even though there is no fee for individual referrals. (D) is wrong because this arrangement does not constitute an improper partnership or association with a nonlawyer for the purpose of practicing law. A lawyer's professional association with a nonlawyer is improper if the nonlawyer: (i) owns an interest in the practice; (ii) is an officer or director of a business involving law practice; or (iii) has the right to control the lawyer's professional judgment. [ABA Model Rule 5.4(d)] None of these is the case here; the bank is acting solely as a referral agent, and has nothing to do with the operation of the lawyer's practice.

An attorney represents a man who has pleaded guilty to a burglary charge. The accused told the attorney during one of their confidential conversations that this is the fourth time he has been busted for burglary-one other time in this state and two times in the neighboring state. The accused could be facing a mandatory 5-10 years in prison as a repeat offender, but he and the attorney are hoping that his pleading guilty will result in a shorter sentence. Unknown to the attorney or the accused, a glitch in the state computer files resulted in a pre-sentencing report that did not pick up either the accused's in-state or out-of-state prior convictions. At sentencing, the judge states, "Normally, I throw the book at young men like you who have no respect for the property of others. However, because I see you have no prior criminal record, I think you deserve another chance. I sentence you to two years' probation and 300 hours of community service." Both the attorney and the accused remain silent. Is the attorney subject to discipline?

The attorney is not subject to discipline for failure to correct the court's mistake because the mistake did not originate with the attorney or his client. In this setting, the attorney would have a duty to speak out if something he or his client had done had given the court a false impression, but otherwise, he cannot breach his duty of confidentiality to the disadvantage of his client. [See ABA Model Rules 1.6(a), 3.3] had the judge asked the attorney directly whether this was his client's first offense, the attorney would have had to respond truthfully or have asked to withdraw (which would also have given the judge his answer). To answer otherwise would be perpetrating a fraud on the court.

An attorney worked at the United States Department of Labor and was responsible for compiling certain corporate safety records into an annual report containing the accident statistics. The report is used internally and in discussions with companies, but it is not distributed to the general public. However, a person may obtain a copy of the report by filing a formal request under the Freedom of Information Act. During the last three years, Company A has had more accidents than any of the other reporting companies. Six months ago, the attorney left the Labor Department and took a job with a private law firm. Recently, a person came to the attorney seeking representation in a suit against Company A for injuries he sustained while working at Company A's factory. The attorney agreed to represent the client. Is the attorney subject to discipline?

The attorney is not subject to discipline for taking the case because the relevant information he obtained while working as a government attorney is not confidential. The general rule is that a government lawyer who receives confidential government information about a person must not later represent a private client whose interests are adverse to that person, if the information could be used to the material disadvantage of that person. [ABA Model Rule 1.11(c)] The rule covers only "confidential" information, which means information that the government is prohibited from revealing or has a privilege not to reveal, and which is not otherwise available to the public. Here, because the information is available under the Freedom of Information Act, it is not confidential. In fact, any attorney representing the client could obtain the information; thus, the attorney is free to use it. (A) is wrong because an attorney is not barred from ever working on a case where he gained any relevant information while working for the government. To bar representation, the information must be confidential. (B) is wrong because this type of consent is required when the attorney takes on a representation in private practice in a matter in which the lawyer participated personally and substantially while in government service. A "matter" is a set of specific facts involving specific parties. Here, the attorney was not involved in any matter while in government service that concerned the client's claim against the chemical company. (D) is wrong because the information is not confidential and thus can be used against the chemical company. Furthermore, even if the information were confidential, mere nonuse would not be sufficient; the attorney would not be permitted to represent the client.

An attorney represented a defendant in a criminal trial. After the jury returned a guilty verdict, the defendant was taken to jail and the jury was discharged. While walking to his car, the disappointed attorney spotted one of the courtroom spectators in the parking lot. The attorney recalled that the spectator had been a member of the jury pool, but he had exercised a peremptory challenge against her because he instinctively felt that she would vote against the defendant. Despite not being selected as a juror, the spectator developed an interest in the case and had attended the entire trial. In an attempt to determine whether his instinct during jury selection was correct, the attorney approached the spectator and asked her whether she would have voted to convict the defendant. The spectator said, "I'd rather not talk about it." When the attorney explained that he was simply looking for constructive feedback, the spectator changed her mind and agreed to a brief interview. The attorney and spectator spoke for a few minutes, and the communication did not involve misrepresentation, coercion, duress, or harassment. Is the attorney subject to discipline?

The attorney is subject to discipline because the spectator initially declined the attorney's request for an interview. ABA Model Rule 3.5(c) provides that after the trial is over and the jury is discharged, a lawyer must not communicate with a former juror or prospective juror if any of the following conditions is met: (i) local law or a court order prohibits such communication; (ii) the juror has told the lawyer that she does not want to communicate; or (iii) the communication involves misrepresentation, coercion, duress, or harassment. Here, the attorney violated the second condition-he persisted with his interview request after the spectator said that she did not want to talk with him. (C) is incorrect. Even though the communication did not involve coercion, duress, or harassment, the attorney still spoke with the spectator after she declined his request, violating the rule. (D) is incorrect because ABA Model Rule 3.5(c) applies to all jurors and even prospective jurors. (B) is too broad. There is no blanket prohibition regarding post-trial contact with jurors and prospective jurors. Rather, such communications are subject to conditions, and the attorney violated one of these conditions.

An attorney and a real estate broker are friends. The real estate broker is of the opinion that the attorney is one of the best real estate lawyers in the community, and she recommends him to those persons seeking an attorney to close real estate transactions. The attorney needs the business and appreciates the real estate broker's recommendations. The attorney has given the real estate broker some of his business cards to give to individuals seeking a real estate lawyer. Currently, the attorney does not pay the real estate broker a referral fee. However, the attorney feels guilty for accepting the much-needed business and giving nothing in return. He has been considering a more formal arrangement whereby he would pay the real estate broker a modest $100 referral fee. In addition, he has been considering taking the real estate broker and her husband on a European vacation with him as a way of expressing his appreciation for her past referrals. Which of the following is correct?

The attorney may represent clients referred by the real estate broker only if he continues his current practice of not paying the real estate broker a referral fee. handing out cards no go - a lawyer generally must not, personally or through a representative, initiate live person-to-person contact with an individual known to need legal services in a particular matter, for the purpose of offering legal services in that matter.

The driver of a car and his passenger were injured as the result of a collision with a bus. They believe the bus driver was entirely at fault, and they want to bring a negligence action against the bus company. They engage in an initial consultation with a local attorney. In the course of the consultation, the attorney realizes that the bus at issue belongs to a bus company that the attorney's firm is representing in an unrelated matter. The attorney interrupts the conversation, explains this potential conflict of interest, and obtains the written consent of both the driver and passenger to represent them. Which of the following is not true? A - The attorney may not represent the car's driver and passenger because he has not obtained the written consent of the bus company. B - The attorney may not represent the car's driver and passenger because he did not inform the passenger that he may have a cause of action against the car driver. C - The attorney may represent the car's driver and passenger. D - The attorney may appear on the bus company's behalf at a court hearing that afternoon.

The attorney may represent the car's driver and passenger. The attorney may not represent both the passenger and the driver when there is a potential conflict of interest between them unless: (i) the attorney reasonably believes that he can represent both clients effectively; and (ii) the passenger and the driver give informed consent, confirmed in writing. The attorney must withdraw from the joint representation, however, if later discovery shows that the passenger has an actual claim against the driver. Here, there is no indication that the driver, the passenger, and the bus company all gave informed consent, confirmed in writing (D) is true because the attorney already represents the bus company in the unrelated matter.

Two years ago, an attorney represented his client when he sold his property. Unbeknownst to the attorney, the client made some fraudulent statements to the buyer about the value of some mineral deposits on the property. The buyer recently discovered the fraud and is now in the attorney's office threatening to immediately file a civil fraud suit against both the client and the attorney. The buyer accuses the attorney of engineering the fraud and helping his client carry it out. The only way that the attorney can convince the buyer that he had no part in the fraud is to tell the buyer a fact that the client disclosed to him in the deepest confidence when he was working on the property transaction. May the attorney disclose the fact without the consent of the client?

The attorney may reveal the confidence even if doing so will subject his client to civil or criminal liability. A lawyer may disclose a client's confidence "to establish a defense to a criminal charge or civil claim against the lawyer based upon conduct in which the client was involved . . . ." [ABA Model Rule 1.6(b)(5)] Although the lawyer must wait until the assertion of misconduct arises, he need not await the filing of a formal charge or complaint. The lawyer may defend himself by responding directly to a third party who has made such an assertion. [See Restatement §64, comment c] (A) is wrong because the lawyer may disclose the fact even if doing so harms the client. (C) is wrong because the lawyer need not wait for the complaint to be filed, as explained above. (B) is wrong because it ignores the self-protection exception to the general rule of confidentiality.

An attorney was appointed by the court to defend a client at his criminal trial for second degree murder. The attorney started interviewing potential witnesses. When she interviewed the client's landlord, the landlord said that on the night of the murder, the client came home very late and was wearing a shirt covered with blood. The landlord died before trial without speaking to state authorities. Which of the following best states what the attorney should do with respect to the information she has learned from the landlord?

The attorney should keep the information in confidence unless the client authorizes her to reveal it, even though the death of the landlord has made it impossible for the prosecutor to obtain the information other than from the attorney. The attorney obtained this information from the landlord in the course of representing her client; therefore, it is subject to the attorney's duty of confidentiality. Absent the consent of the client, an attorney must not reveal any information relating to the representation of the client. [ABA Model Rule 1.6] (A) is incorrect because an attorney has no ethical obligation to reveal harmful facts, and, in fact, may be disciplined for doing so. (B) is incorrect because, as noted above, there is no duty to reveal this information; thus, there is no obligation to urge the client to reveal the information or withdraw. (D) is incorrect because this information is confidential. The ethical duty of confidentiality covers more kinds of information than the attorney-client privilege, which covers only confidential communications between the attorney and client. The ethical duty of confidentiality covers any information the attorney obtains relating to the representation of the client, no matter what the source of the information.

For many years, a tax attorney has handled all of the tax work for his client, a sculptor. One evening, the sculptor invited the attorney to his studio to discuss some tax returns that had to be filed the next day. In the studio, the attorney saw a small sculpture that would be perfect for his office. At the close of their tax discussion, the attorney offered to buy the sculpture for $10,000, its approximate fair market value. The sculptor told the attorney that it was not for sale. In due course, the attorney sent the sculptor a bill for a $750 fee for the tax work. A few days later, the small sculpture was delivered to the attorney's office with a note from the sculptor, saying that he hoped the sculpture would satisfy the recent bill, and he wanted the attorney to have the sculpture as a token of his gratitude for the excellent tax advice. Would the attorney be subject to discipline for accepting the small sculpture from the sculptor?

The attorney would not be subject to discipline for accepting the sculpture because he did not solicit the gift. Although ABA Model Rule 1.8(c) prohibits a lawyer from soliciting a substantial gift from a client when the lawyer is not related to the client, it does not prohibit a lawyer from accepting an unsolicited gift from a client, even if the gift is substantial (although the gift may be voidable for undue influence). Moreover, comment 6 to ABA Model Rule 1.8 states that a lawyer may accept a gift from a client if the transaction meets general standards of fairness. Here, the attorney did not solicit the gift, and there are no facts to suggest undue influence or unfairness. Thus, the gift is proper. (A) is wrong because it is too broad. A lawyer may accept a gift of substantial value from a client if the conditions stated above are satisfied. (B) and (D) are wrong because the value of the attorney's recent work and the value of the sculpture are irrelevant. The attorney did not charge more than the $750. In addition to discharging the $750 fee bill, the sculptor obviously intended to make a gift to the attorney in gratitude for years of work in the past.

An attorney has just opened an office in a town where he does not know many people and has few contacts. The attorney has just heard that a group of townspeople, including a former client, have been arrested and are being held at the county jail for conducting a noisy demonstration outside the local high school to protest an impending strike by the teachers. The attorney strongly believes that the arrests were politically motivated and that the demonstrators have been deprived of their freedom of expression in violation of the First Amendment. He also realizes that he might gain some favorable public exposure if he were to help the arrestees. He therefore goes down to the county jail and offers his legal services, free of charge, to any of the arrested demonstrators who want legal assistance. Are the attorney's actions proper?

The attorney's actions are proper because he offered his services free of charge. Generally, a lawyer is prohibited from seeking employment by initiating live person-to-person contact with a person known to need legal services in a particular matter. However, this prohibition applies only when "a significant motive" for the solicitation is the lawyer's pecuniary gain. [ABA Model Rule 7.3(b)] Thus, a lawyer who volunteers to represent someone free of charge is not subject to discipline for solicitation. (A) is incorrect because, as discussed above, it is not necessary for the attorney to have previously represented any of the demonstrators in order for his actions to have been proper. (C) is incorrect because, as discussed above, this situation falls within an exception to the ban on live person-to-person solicitation. (D) is incorrect because the attorney's actions are proper even if he is motivated by a desire to obtain publicity, provided that this is not a substantial motive for his offer.

An attorney is defending her client in a civil fraud case in which it is relevant to know what advice the client received in confidence from an independent certified public accountant. The jurisdiction has no evidentiary privilege for confidential communications between accountants and their clients. The accountant telephoned the attorney and asked how he should respond to the plaintiff's lawyer's request to speak with him privately about the case. Reasonably believing that the accountant would not be harmed by refusing to talk informally with the plaintiff's lawyer, the attorney responded that if the plaintiff's lawyer subpoenaed him to testify, then he must do so, but encouraged him not to talk to the plaintiff's lawyer about the case unless under subpoena. Was the attorney's advice to the accountant proper?

The attorney's advice to the accountant was proper because the accountant acted as the client's agent in rendering accounting advice. An attorney may request that someone other than a client refrain from voluntarily giving relevant information to another party if the person is a relative or agent of the client and the attorney reasonably believes that the person's interests will not be adversely affected by refraining from giving the information. [ABA Model Rule 3.4(f)] Here, the attorney reasonably believed that refusing to talk informally with the plaintiff's counsel would not harm the accountant, and the accountant was the client's agent in rendering the accounting advice. (A) is wrong because it is the accountant's agency, not privilege, that affects the propriety of the attorney's advice. The lack of an evidentiary privilege does not give the plaintiff's counsel a right to talk to the accountant informally if the accountant chooses not to. (B) is wrong because the attorney did not interfere with the plaintiff's access to evidence; if the plaintiff wants to know what the accountant has to say, he can simply take the accountant's deposition. (D) is wrong because it was not improper for the plaintiff's counsel to attempt to talk informally with a third-party witness such as the accountant.

An attorney limits her law practice to the representation of plaintiffs in actions for medical malpractice. She has developed a standard employment contract to use with all clients who desire a contingent fee arrangement. The contract requires the client to pay in advance the first $500 of litigation expenses (which is a reasonable sum in the ordinary medical malpractice case), and states that the attorney's fee shall be 20% of any amount recovered without going to trial, 30% of any amount recovered if the case goes to trial, and 35% of any amount recovered if the case is appealed. A client signed the attorney's standard form contract, and after putting in only 10 hours' work on the client's case, the attorney was able to work out a settlement agreement in which the client received $10,000 from the defendant's malpractice insurance carrier. The client was delighted with the settlement, but he is unhappy with having to pay 20% of it ($2,000) to the attorney for so few hours' work. With respect to the clause of the attorney's standard form contract that requires the client to pay in advance the first $500 of litigation expenses, which of the following is correct?

The clause is proper. There is nothing in the ABA Model Rules that prohibits a lawyer from requiring a client to pay in advance the first $500 of litigation expenses, so long as the $500 payment is reasonable. [ABA Model Rule 1.5(a)] B - The attorney must delete the clause because it is an effort to impose a uniform provision on all clients, irrespective of their particular needs and situations. (B) is incorrect b/c Rules do not suggest that every clause of a standard form employment contract must be individually tailored to each client.

A plaintiff brought a civil action to recover damages for personal injuries he suffered as the victim of alleged police brutality inflicted by three defendant police officers. The trial was widely reported by the media. The jury returned a verdict in favor of the plaintiff and against the three police officers for $500 million. When the trial judge received the verdict, he was shocked by the size of the award. Before dismissing the jurors, the judge told the jurors that when they were sworn in, they had promised that they would deliver a verdict based on the evidence and that they would not be swayed by passion or prejudice. The judge further admonished the jurors that they had failed in those duties, that they had made a mockery of justice, and that they should be ashamed of themselves. He then dismissed the jury, and the defense lawyers renewed their motion for judgment as a matter of law and, alternatively, moved for a new trial. The judge announced that he would rule on the motions the following Monday at 10 a.m. in open court. The press reports of the verdict and the judge's comments to the jury created a great public tumult in the city where the case was tried. On the following Monday, the courtroom was jammed with reporters. Primarily for the purpose of educating the reporters, the judge first gave a detailed explanation of the legal requirements for granting a renewed motion for judgment as a matter of law and for granting a new trial motion. He then granted the renewed motion for judgment as a matter of law and, alternatively, the motion for a new trial. Were the judge's actions proper?

The communication with the reporters was proper, but the statements made to the jury were not. A judge should not "commend or criticize jurors for their verdict other than in a court order or opinion in a proceeding." A judge's commendation or criticism may impair a juror's ability to be impartial in a subsequent case. [CJC Rule 2.8(C)] The judge's statements to the jury clearly violate this Rule; thus, (A) and (C) are incorrect. Although judges should not comment on pending cases in a manner that might interfere with fairness, judges are not prohibited from making public statements in the course of their official duties or from explaining for public information the procedures of the court. [CJC Rule 2.10(D)] The judge's explanation of the motions to the reporters was an appropriate way to inform the public of the meaning and significance of the judge's decision to reject the $500 million verdict; thus, (B) and (C) are incorrect.

A worker sued his employer, claiming that he was permanently and totally disabled due to a back injury he suffered on the job. The employer's lawyer strongly suspected, but had no proof, that the worker continued his hobby of skydiving after the alleged back injury. In due course, the employer's lawyer met with the worker's lawyer for a settlement discussion. The employer's lawyer told the worker's lawyer that they had movies of the worker jumping out of an airplane two weeks after his purported injury. The worker's lawyer excused herself to make a telephone call to the worker. When she asked the worker whether he had been skydiving after the accident, he admitted that he had. With the consent of their respective clients, the two lawyers then settled the case for $400. Is the employer's lawyer subject to discipline?

The employer's lawyer is subject to discipline because he lied about having movies. When dealing on behalf of a client with a third person, a lawyer must not knowingly make a false statement of law or material fact. [ABA Model Rule 4.1(a)] The employer's lawyer knew that he had no movies; his statement to the worker's lawyer was a bald lie. (A) is wrong because the employer's lawyer's statement was a knowing misrepresentation of material fact, not the kind of puffery that is tolerated in settlement negotiations. [See comment 2 to ABA Model Rule 4.1] (B) is wrong because the ends do not justify the means. (C) is wrong because the employer consented to the $400 settlement. Given the employer's risk of going to trial in a case where the plaintiff is claiming total and permanent disability due to a back injury, the small settlement was not unreasonable.

Continuously since 1910, the law firm of Alpha & Beta has practiced under that name. The founders of the firm are long dead. No partner named Beta now practices with the firm. Two partners named Alpha were practicing with the firm, but one recently left because she was appointed to the state supreme court. May the firm continue to use the name Alpha & Beta?

The firm may continue to use the name Alpha & Beta if it is not misleading. A firm may practice under a trade name, provided that the trade name is not misleading. [Comment 5 to ABA Model Rule 7.1] (A) is wrong because a firm may continue using the name of a deceased partner. [Id.] (B) is wrong because the person who left was not a name partner. Generally, when a name partner enters public service and is not in private practice for a substantial period, the firm must cease using that person's name. [Comment 8 to ABA Model Rule 7.1] Here, the Alpha surname refers to the deceased founder and not the partner who was appointed to the state supreme court. If, however, the use of the Alpha name in the firm name would mislead potential clients (e.g., by making them think they could gain an advantage in the state supreme court by hiring that firm), then continued use of the name would violate the ethics rules. [See ABA Model Rule 7.1] (D) is wrong because the rules on firm names are subject to the more general provisions on misleading communications.

An attorney is admitted to practice only in State A, where he specializes in securities and real estate finance law. In that role, the attorney advised his client that the law of State B did not require the client to include information about certain mineral rights in a disclosure statement that the client had to file in State B in order to sell some real estate limited partnership interests to State B citizens. Acting on the attorney's advice, the client did not disclose the information and did sell partnership interests to State B citizens. Later, the attorney became a full-time trial court judge in State A. Later still, State B brought a criminal action against the client for failing to disclose the mineral rights information in his State B disclosure statement. One of the client's defenses is that he lacked the necessary criminal intent because he was acting in good faith based on the advice of his counsel, the attorney who is now a judge. The client needs the judge's testimony to prove that the judge did indeed advise him that he was not required to disclose the mineral rights information. The judge, in State A, is beyond the subpoena power of the State B court. May the judge voluntarily testify on behalf of the client?

The judge may testify because he is testifying to facts, not the defendant's character. CJC Rule 3.3 prohibits a judge from testifying voluntarily as a character witness, but it says nothing about serving as an ordinary fact witness. (A) is wrong because there is no such rule. Judges are not disqualified from testifying in criminal cases. (B) is wrong because an attorney need not be admitted to practice in State B in order to advise a client about State B law. Even if that were untrue, the judge's testimony would still be admissible as evidence of the client's lack of criminal intent. (C) is wrong because it states a nonexistent rule. While a judge is not competent to be a witness at a trial over which he himself is presiding [see Fed. R. Evid. 605], no rule forbids a judge from serving as an ordinary fact witness in a case that is pending before a different judge in his own court or a court that is under his court's appellate jurisdiction.

The mother of a full-time trial judge owns a small business that she wishes to sell. After she and a prospective buyer come to terms on the sale, the buyer has his lawyer draw up a sales contract and presents it to the judge's mother, who asks her son, the judge, to review it for her. The judge agrees, marks up the contract, and returns it to his mother to present to the buyer's attorney. The judge's mother did not tell the buyer that her son reviewed the contract. The buyer has no known reason for being likely to appear in the judge's court. Were the judge's actions proper?

The judge's actions were proper because he did not charge his mother a fee. Although a full-time judge may not practice law, there is an exception for this type of transaction. A judge may, without compensation, give legal advice to, and draft and review documents for, a member of the judge's family. [CJC Rule 3.10] (A) is wrong because whether the buyer might appear in the judge's court does not affect the judge's ability to prepare documents for his mother. If the buyer does appear in his court, the judge's participation in that proceeding will be evaluated at that time in light of the facts. (C) is wrong because, as stated above, there is an exception for reviewing documents for relatives without compensation. (D) is wrong because, although the judge cannot act as a negotiator, there is no requirement that his identity as the person who made the revisions be kept a secret.

A state has a so-called three strikes law that is designed to give life sentences to career felons. When a person is convicted of her third felony, the three strikes law forces the sentencing judge to give the person a life sentence, even if the third felony does not involve violence, and even if the maximum punishment for the third felony by itself would be as little as two years in state prison. Many judges in the state believe that the three strikes law is unconscionably harsh and should be repealed. The state's legislature, however, has refused to act. A citizen group was successful in qualifying a voter initiative to abolish the three strikes law. Many judges have spoken out publicly in favor of the voter initiative. A lawyer who has both an accounting degree and a law degree earns most of her living by serving as a court-appointed master in complex cases where her dual skills are useful. The bulk of her service is in a court that is dominated by judges who favor the voter initiative. For the sole purpose of getting more court appointments from those judges, the lawyer has contributed 200 hours of her time to the campaign in favor of the voter initiative. Which of the following statements is true? The judges did not act properly in speaking out in public in favor of the voter initiative. The judges are subject to discipline for speaking out in public in favor of the voter initiative. The lawyer acted properly in contributing her time to the campaign in favor of the voter initiative. The lawyer is subject to discipline for contributing her time to the campaign for the purpose of getting more court appointments

The lawyer acted properly in contributing her time to the campaign in favor of the voter initiative. Lawyers are permitted to participate in the political process, except when the law or a legal ethics rule prohibits it. The legal ethics rule nearest to the point is ABA Model Rule 7.6, which prohibits a lawyer from making a political contribution for the purpose of securing judicially appointed legal work. That rule does not apply here for two reasons. First, the rule does not apply to voter initiatives and referendums, and second, the rule does not apply to contributions of uncompensated services. [Comment 2 to ABA Model Rule 7.6] Choice (A) is not true. CJC Rule 3.1 allows judges to engage in extrajudicial activities that are not otherwise prohibited. Comment 1 to CJC Rule 3.1 encourages judges to speak out publicly on issues concerning the law, reasoning that judges are uniquely qualified to engage in activities that concern the law, the legal system, and the administration of justice.

A lawyer is interested in obtaining legal business from a mineworkers' union that has many members in the state in which the lawyer practices. As a result of a recent mine fire and explosion in which several union members were killed, the union has succeeded in persuading the appropriate state agency to bring an administrative action against the company that owns the mine for failing to install smoke detectors, which might have saved some lives in the disaster. Although the lawyer is in no way involved in the case, he sees this as an opportunity to obtain future business from the union by showing the union that he is strongly on its side in the mine disaster case. The lawyer telephones a popular call-in radio show, and says that he is shocked and appalled at the callousness of the mining company that caused the recent disaster in which so many miners were killed. He further opines that the mining company was willful and wanton in its failure to install smoke detectors, and expresses hope that the company will not be allowed to escape the consequences of its despicable conduct. Without the lawyer's knowledge or consent, his statement was later printed in several news-papers in the state. Is the lawyer subject to discipline for his conduct?

The lawyer is not subject to discipline because he did not make any false or misleading claims about himself or his services. Lawyers, like other citizens, have the right to express their views in the media on newsworthy issues. Even if a lawyer's sole purpose in seeking media publicity is to lure clients, the state may not impose professional discipline on the lawyer absent a compelling state interest. A lawyer who uses the media to lure clients may, however, be disciplined for making statements or claims that are false or misleading about the lawyer or his services. [ABA Model Rule 7.1] Here, the lawyer made no statements about himself or his services other than the fact that he is a lawyer and his opinion about the incident. There is nothing false or misleading in his communication. (A) is wrong because, as discussed above, the fact that the lawyer was motivated by the desire to attract fee-paying business is irrelevant. This is not a case of live person-to-person solicitation, which is the only circumstance in which this consideration is relevant. (B) is wrong because it is overbroad. Lawyers can and do make public statements about pending litigation all the time. Lawyers who are involved in a proceeding cannot make statements that they know will have a substantial likelihood of materially prejudicing an adjudicative proceeding. [ABA Model Rule 3.6(a)] That is not the case here; the lawyer has no reason to believe his opinion will materially prejudice the state agency bringing the action. (D) is wrong because the fact that the statement was printed in the newspapers is of absolutely no consequence; it does not affect the propriety of the lawyer's behavior regardless of whether he gave consent.

The Department of Children's Services ("DCS"), a state agency, has removed a woman's children from her home and is attempting to terminate her parental rights. The woman retains a lawyer to fight DCS's actions. Prior to the first hearing on the matter, a lawyer from DCS contacted the woman's brother to set up an interview. The lawyer told the brother that he wanted to talk to him about his sister, and that the interview would last for about one hour. The brother, who is childless, had never had any contact with DCS before, and did not know what his duties were. He told the lawyer he would get back to him. The brother then called his sister's lawyer. He told her that he does not want to talk to DCS because he is afraid he might inadvertently say something that will hurt his sister's case. He asked his sister's lawyer how he should proceed. She advised him simply to refuse the interview, because he was not required to talk to them and his sister would be better off if he did not. Is the lawyer subject to discipline?

The lawyer is not subject to discipline because of the brother-sister relationship. A lawyer may advise a person not to voluntarily give information to an opponent or other party if the person is a relative of the client and that person's interests will not be harmed by not volunteering the information. [ABA Model Rule 3.4(f)]

A lawyer represents a defendant in a drug smuggling case. The defendant is in pretrial custody in a distant city and cannot be reached by telephone. One key issue in the defendant's case is on the cutting edge of search and seizure law, and the lawyer believes that he needs help to deal with the issue competently. The lawyer's former law professor is a nationally known expert on search and seizure law, but is not licensed to practice law in that state. The lawyer calls his former professor to ask for help and also asks that the professor keep their conversation confidential. To frame the issue accurately, the lawyer tells the law professor some information that the defendant revealed to the lawyer in confidence. The lawyer does not tell the professor the name of his client. Is the lawyer subject to discipline for disclosing the defendant's confidential information to the professor?

The lawyer is not subject to discipline for disclosing the defendant's confidential information to the professor because the disclosure was necessary to effectively carry out the representation. Unless a client has specifically instructed the lawyer to the contrary, a lawyer can reasonably assume that he has implied authority from the client to disclose confidential information when necessary to carry out the representation. That is particularly true in cases such as this one-where the lawyer cannot easily communicate with his client. [ABA Model Rule 1.6(a)] (A) is wrong because a lawyer may seek advice from an expert without the expert being licensed in the jurisdiction; and in any case, it does not affect the confidentiality rules. (B) is wrong because specific authorization is not required; it may be implied. (D) is wrong because refraining from revealing the client's name is not sufficient to permit revelation of confidential information. There must be an exception to the confidentiality rules or authorization by the client.

When a lawyer was an associate at a law firm, she did the legal work for one of the firm's clients on a land sale transaction that earned the client millions of dollars. In gratitude, the client asked the lawyer whether she had any unfulfilled wishes. The lawyer told the client that she wished she had enough money to start her own solo law practice. The client then told her that he would lend her $100,000 to set up her new practice. In return, she would thereafter do all of his legal work at a 5% discount from her normal hourly fee, and she would pay off the $100,000 loan by monthly payments equal to 10% of her net income for the prior month. The lawyer was delighted. She drafted a complete, detailed agreement between herself and the client, and she advised the client in writing to obtain outside legal advice before signing the agreement. The client obtained the outside advice and signed the agreement, and the lawyer set up her solo practice accordingly. Is the lawyer subject to discipline?

The lawyer is not subject to discipline unless she allows the client to interfere with her judgment in handling other clients' matters. [See ABA Model Rule 5.4] (B) is wrong because there is no law or disciplinary rule that requires the lawyer to notify the law firm regarding the transaction. (C) is wrong because no law or disciplinary rule requires the law firm's consent to the client's leaving the firm and giving his business to the lawyer as a solo practitioner. (D) is wrong because the loan payback clause does not violate the rule against splitting a legal fee with a nonlawyer. [ABA Model Rule 5.4(a)] True, the clause does measure the monthly payments as a percentage of the lawyer's net income in the prior month and most of her net income will probably come from legal fees. However, it makes sense to tailor her loan payments to her income, and the arrangement does not invite the evil that the no-splitting rule was designed to prevent-interference with the lawyer's professional judgment.

A concerned environmentalist hired a lawyer to obtain preliminary and permanent injunctions against a highway construction project that would require draining and filling certain wetlands inhabited by migratory waterfowl. The lawyer is the nation's leading expert in wetland preservation law, and he charges $400 per hour for his services. The environmentalist agreed to pay him at that rate. She gave him a $40,000 advance on attorneys' fees and a $5,000 advance to cover future litigation expenses. The lawyer deposited the entire $45,000 in his client trust account. The lawyer then spent 80 hours preparing and filing a complaint and preparing and arguing a motion for a preliminary injunction. He paid a court filing fee of $50, plus $1,950 in witness fees to wetlands experts who testified at the hearing on the preliminary injunction motion. The judge denied the preliminary injunction motion. The lawyer sent the environmentalist a bill for $32,000 in attorneys' fees and $2,000 in litigation expenses, and he told her he would deduct those sums from the advances she had given him unless he heard from her to the contrary within 15 days. In light of the loss of the preliminary injunction motion, the environmentalist was outraged at the size of the lawyer's fee; she immediately fired him and demanded the prompt refund of her entire $45,000. Which of the following amounts must the lawyer promptly refund to his former client, the environmentalist?

The lawyer must promptly refund $11,000 because that amount is not in dispute. The lawyer claimed $32,000 in fees (80 hours at $400 per hour) from the $40,000 fee advance (leaving an excess of $8,000). The lawyer further claimed $2,000 from the $5,000 advance for expenses (leaving an excess of $3,000). Adding $8,000 and $3,000 produces an $11,000 refund due immediately from the lawyer to the environmentalist. [ABA Model Rule 1.16(d)] The environmentalist apparently disputes the lawyer's right to $32,000 in fees; thus, that disputed amount must remain in the lawyer's client trust account until the fee dispute is settled. [ABA Model Rule 1.15(e)] (A) is wrong because when a lawyer is fired or withdraws, he must immediately refund the unspent portion of the expense advance and the portion of the fee advance that he does not claim to have earned. [ABA Model Rule 1.16(d)] (C) is wrong because the lawyer may retain the disputed portion of the expense advance, as explained above. (D) is wrong because the lawyer is entitled to retain the disputed $32,000 in his client trust account until the fee dispute is settled.

An author wrote a best-selling novel based on the life and crimes of John Dillinger, the famous bank robber. The author sold the movie rights to a film producer, who promised to pay the author a lump-sum royalty of $5 million upon the release of the movie. After the producer hired an actor to play the lead role and made other expensive preparations for filming, the author repudiated the contract. The producer hired a lawyer to sue the author for a declaratory judgment that the contract was valid and enforceable. At the producer's request, the lawyer agreed to do the legal work on a contingent fee basis: If the producer wins, the lawyer will be paid 1.75% of the gross receipts from the movie, but if the producer loses, the lawyer will be paid nothing. The producer and the lawyer entered into a written fee agreement that contains all the details required by the rules of legal ethics. Which of the following statements is true?

The lawyer's fee agreement is proper even though it gives the lawyer a personal interest in the subject of the litigation. The lawyer has acquired a personal interest in the movie, which is in one sense the real subject of the litigation. However, the rule against acquiring a personal interest in the subject of litigation has an exception that allows a lawyer to represent a client for a contingent fee. [ABA Model Rule 1.8(i)(2)] (A) is wrong because the rule on literary rights contracts covers only literary works based in substantial part on information relating to the representation. Here, the movie concerns John Dillinger, not the producer. [See ABA Model Rule 1.8(d)] (B) is wrong because, as discussed above, a lawyer may acquire an interest in the subject matter of the litigation in the form of a contingent fee. (C) is wrong because the fee agreement between the producer and the lawyer does not require the author's informed consent. The author is not a current or former client of the lawyer, and the author has no apparent interest that would cause the lawyer a conflict and force him to disclose and explain the material risks and available alternatives and obtain the author's consent.

A corporation is named as the defendant in an employment discrimination suit. The corporation's attorney has scheduled the taking of a deposition of one of the corporation's employees who is in poor health, as a precaution in case the employee should die before trial. The employee is not an officer or shareholder in the corporation, and he is not a party to the lawsuit. He is in charge of the corporation's personnel department, and he is responsible for ensuring that the corporation's hiring practices comply with the laws against employment discrimination. Without seeking the consent of the corporation's attorney, or even telling him, the plaintiffs' attorney had lunch with this employee several days before the deposition, and on that occasion the plaintiffs' attorney pumped the employee for information relevant to the lawsuit. When the corporation's attorney learned what had happened, he telephoned the plaintiffs' attorney and called him a slimy, mud-sucking sneak. Who is subject to discipline and why?

The plaintiffs' attorney is subject to discipline because he should not have talked with the employee about the case without the consent of the corporation's attorney. (A) is incorrect because lawyers should treat all participants in a proceeding with courtesy, respect, and cooperation. [See Restatement §106] (D) is incorrect because there is nothing that prohibits a lawyer from talking with a deposition witness about the case before a deposition. However, a lawyer must not communicate about a case with a person he knows to be represented by a lawyer, without first getting the lawyer's consent. [ABA Model Rule 4.2] Thus, (C) is correct and (D) is incorrect. Also, Comment 7 to ABA Model Rule 4.2 states that, in the case of a represented organization, a lawyer for one party may not communicate concerning the matter with persons whose acts or omissions in connection with the matter may be imputed to the organization for purposes of civil or criminal liability. Here, the corporation's employee fits the description of such a person. Thus, (B) is incorrect.

A man was charged with first degree murder. It is claimed that he captured his victim, dragged her into dense woods, and stabbed her with a knife. The accused has pleaded not guilty. During the prosecution's case-in-chief, one of the prosecutor's witnesses testified that he had seen the accused's car near the scene of the crime shortly before the murder. This courthouse has no private room set aside for conferences between defendants and their counsel. Thus, at the next recess, the accused and his attorney held a hurried, whispered conference in the hallway, during which the attorney asked the accused why he had not told him about driving around that area. The accused replied that he had not realized that anyone had seen him. Unbeknownst to the accused and his attorney, the prosecutor was standing nearby and overheard their whispers. Which of the following statements is correct?

The prosecutor must ignore what she heard and proceed with the case in the normal manner.

A public defender was assigned to represent a defendant at the defendant's preliminary hearing on a charge of kidnapping for ransom. Against the public defender's advice, the defendant testified on his own behalf at the preliminary hearing. The defendant was bound over for trial. At that point, the defendant's brother provided money to hire a private lawyer to represent the defendant, and the public defender was discharged. The defendant testified on his own behalf at the trial, and the matter concluded after the jury acquitted him. Later, in connection with his work on another matter, the public defender read the transcript of the defendant's trial. Based on information the public defender learned while representing the defendant, the public defender concluded that the defendant had committed perjury, both at the preliminary hearing and at the trial. However, the public defender does not believe that the defendant poses a danger to the community. May the public defender reveal the defendant's perjury?

The public defender may not reveal the defendant's perjury because to do so would violate the public defender's duty of confidentiality. No exceptions to the confidentiality requirement apply to these facts. There is no indication that revealing the perjury is necessary to prevent reasonably certain death or substantial bodily harm. Also, there is no indication that the defendant's perjury is a crime that is reasonably certain to result in substantial injury to the financial interests of another, in furtherance of which the defendant has used the public defender's services. The obligation to reveal perjury under the Model Rules does not apply because that obligation ceases at the end of the proceedings, and both proceedings here have concluded. (A) is wrong because the proceeding has ended and, therefore, the public defender is obligated to keep the information in confidence. (B) is wrong for the same reason. Furthermore, even if the trial was ongoing, the public defender would not be obligated to reveal the perjury because he was not representing the defendant at trial. The public defender represented the defendant in the preliminary hearing, which has concluded, ending his obligation to disclose. (C) is wrong because what the public defender believes about the defendant's dangerousness is not relevant. There is no exception to the duty of confidentiality based on the client's violent propensities.

Lawyer Larry currently works for the New York City Landmarks Commission. He represents the Commission, a municipal entity, in landmark-related disputes, including a number of disputes with Blackacre Real Estate Company. Larry oversaw all aspects of the litigation against Blackacre and personally appeared in court on behalf the Commission. Impressed with Larry's skills, Blackacre approached Larry to join the company as general counsel and Larry accepted Blackacre's offer. Is Larry subject to discipline?

The rule is that a lawyer who is an attorney for the government may be subject to discipline for negotiating for private employment with an entity that is involved as a party in a matter for which in which the attorney is participating personally and substantially. Note that there is a special exception for judges' and adjudicative officers' law clerks who are seeking work after their clerkships end. Thus, Larry is subject to discipline because Larry negotiated private employment with an entity that is involved as a party in a matter in which Larry is participating personally and substantially. Larry's own involvement in the matter is material. Although Larry's employment with the Commission may be at will, he is still subject to discipline. It does not matter that Blackacre approached Larry.

A solo practitioner who does municipal bond work in a single state is nearing retirement age and takes on a young attorney as a partner. Their partnership agreement provides that the solo practitioner will train the young attorney and will receive 75% of the partnership's net earnings during the first three years, and that the young attorney will receive the remaining 25%. The agreement also provides that if the young attorney leaves the partnership before the end of the first three years, he will remit to the solo practitioner 75% of all fees he earns thereafter from municipal bond work in the state. Finally, the agreement provides that if the solo practitioner and the young attorney are still partners when the solo practitioner retires, the young attorney will pay the solo practitioner retirement benefits of $3,000 per month until the solo practitioner's death; in return, upon his retirement, the solo practitioner will turn over to the young attorney all of the partnership assets (including goodwill) and will not thereafter practice municipal bond law in the state. Are the solo practitioner and the young attorney subject to discipline for entering into this partnership agreement?

The solo practitioner and the young attorney are subject to discipline for entering into the partnership agreement because of the restriction on the young attorney's right to practice. A lawyer must neither make nor offer a partnership agreement that restricts a lawyer's right to practice after termination of the relationship, except for an agreement concerning benefits upon retirement. [ABA Model Rule 5.6(a)] Under the agreement here, if the young attorney leaves the partnership within the first three years, he must pay the solo practitioner 75% of the fees he earns thereafter from municipal bond work he does in the state. This provision unduly restricts the young attorney's right to practice. (A) is wrong because it overlooks the improper restriction on the young attorney's right to practice. (B) is wrong because it overlooks the restriction on the young attorney. Furthermore, it implies that the sale of assets in exchange for the restriction on the solo practitioner is necessary to restrict the solo practitioner's right to practice; it is not. (C) is wrong because the restriction on the solo practitioner's right to practice is properly incident to the retirement benefits he will receive from the young attorney.

A steel company merged with an iron corporation. The state attorney general sued the steel company and the iron corporation in federal court to enjoin the merger, alleging that it was in violation of the federal antitrust laws. The federal district judge enjoined the merger, and the steel company appealed the judge's decision. The steel company's lawyer, doing the legal research for the appeal, found a recent merger decision rendered by the Federal Trade Commission ("FTC") that is directly adverse to the steel company's position. FTC decisions do not control in the United States Courts of Appeal, but they are persuasive. The attorney general failed to cite the FTC decision. Must the steel company's attorney disclose it to the court?

The steel company's lawyer need not disclose the FTC ruling because the court need not follow the decision. An attorney can be disciplined for failing to cite the court to legal authority that is "directly adverse" to the client's position and is from the "controlling jurisdiction." [ABA Model Rule 3.3(a)(2)] Although the decision is directly adverse to the steel company's position, the facts state that FTC decisions do not control in the United States Courts of Appeal. Thus, the decision is not from the "controlling jurisdiction." (A) is wrong because in order to invoke the disclosure rule, the decision must be from a controlling jurisdiction, not merely persuasive. (B) is wrong because it states only one portion of the test that triggers the disclosure rule. While true that the decision is adverse, it is not from a controlling jurisdiction, and thus need not be revealed. (C) is wrong because although the Rule as stated is true, in this case we are not concerned with harmful facts. The issue is whether the steel company's attorney must reveal harmful law.

A family-owned chemical company receives information that some of its sales associates may have entered into price fixing agreements in violation of federal antitrust laws. The shares of the chemical company are not publicly traded and the company is not subject to the jurisdiction of the Securities and Exchange Commission. The general counsel of the corporation hires an outside antitrust lawyer to investigate the matter. The antitrust lawyer and her associate investigated the matter and discovered that some of the sales associates had indeed entered into agreements that could make the corporation civilly and criminally liable under the antitrust laws. The antitrust lawyer reported these findings and her antitrust advice in a confidential letter addressed jointly to the company's chief executive officer and the general counsel. The general counsel wrote back, asking the antitrust lawyer to stand by to defend the company if needed. Months went by, and the antitrust lawyer heard nothing more. Her associate grew restless, and without telling the antitrust lawyer, he told a friend in the Justice Department what the company's salespeople had done. The Justice Department began a price fixing investigation of the company and its competitors. Which of the following is correct?

This question is governed by ABA Model Rule 1.13, not by the SEC's regulations under the Sarbanes-Oxley Act, because the company is not publicly owned and is not subject to the jurisdiction of the SEC. The associate is subject to discipline for tipping off the Justice Department because he violated the duty of confidentiality imposed by ABA Model Rule 1.6(a). When the antitrust lawyer and the associate investigated the price fixing rumor at the request of the company's general counsel, they were operating under ABA Model Rule 1.13(d), which applies to lawyers who are hired "to investigate an alleged violation of law" or to "defend an organization" or its people against a claim arising out of an alleged violation of law. That means that ABA Model Rule 1.13(c) does not apply, and they must not report to outsiders about what they find. Both parts of (A) are wrong-the antitrust lawyer acted properly, and the associate is subject to discipline, as explained above. (B) is wrong because the associate's conduct was not proper. Moreover, the associate probably committed legal malpractice when he tipped off the Justice Department, and the antitrust lawyer might be vicariously liable for his malpractice because she was his supervisor. (D) is wrong because the associate is subject to discipline for violating ABA Model Rule 1.6(a).

In the federal courts and the courts of some states, when a litigant claims that a document is protected by the attorney-client privilege, the trial judge can order the litigant to produce the document for in camera inspection (i.e., private, confidential inspection by the judge) so that the judge can decide whether the document really is privileged. The courts in one state follow the opposite rule: a trial judge must not force a litigant to produce a privileged document for in camera inspection; the trial judge must use the surrounding circumstances and whatever other evidence is available when deciding whether the privilege claim is valid. In a federal case where jurisdiction is based only on diversity of citizenship, the federal judge must apply the privilege law that would be applied by a state court in the state where the federal court sits. A federal district judge was presiding in a diversity case in the state. The defendant's lawyer refused to produce a particular letter from defendant's outside patent counsel to defendant's president. The defendant's lawyer asserted the attorney-client privilege on behalf of her client. The judge glowered down at the defendant's lawyer and ordered her to produce the letter for in camera inspection so that he could see whether the letter was about a legal matter or some nonprivileged subject. The defendant's lawyer would not allow the judge to see the letter, referring him to the clear law of the state in which the court sat. The judge growled and ordered her to hand him the letter. Which of the following is the proper response for the defendant's lawyer to make?

To respectfully ask the court to stay the order to allow time to consult with the client and to seek an interlocutory appeal or extraordinary writ if necessary. Comment 15 to ABA Model Rule 1.6 explains what a lawyer should do when push comes to shove with a judge about a privilege issue: "A lawyer may be ordered to reveal information relating to the representation of a client by a court or another tribunal or government entity claiming authority pursuant to other law to compel disclosure. Absent informed consent of the client to do otherwise, the lawyer should assert on behalf of the client all nonfrivolous claims that the order is not authorized by other law or that the information sought is protected against disclosure by the attorney-client privilege or other applicable law. In the event of an adverse ruling, the lawyer must consult with the client about the possibility of appeal . . . . Unless review is sought, however . . . [the lawyer may] comply with the court's order." The attorney-client privilege belongs to the defendant, not to the defense lawyer, and the defense lawyer should not waive it without talking over the options with her client. [See comments 15 and 16 to ABA Model Rule 1.6]

A client hired an attorney to put together a complex real estate syndicate. In connection with that work, the client disclosed to the attorney a great deal of confidential information about the client's financial affairs. When the task was about half completed, the attorney's wife was killed in a car accident and his family's house burned down, all in the same week. The attorney was so emotionally and physically drained that he felt he could not competently continue with the work for his client. The client refused to allow the attorney to withdraw. The attorney begged the client to allow him to turn the files over to his law partner, an excellent real estate lawyer who was completely trustworthy and perfectly competent to handle the matter. The client refused to allow his files to be turned over to any other lawyer and insisted that the attorney himself promptly complete the work. What should the attorney do?

Withdraw and turn the client's files over to the client. A lawyer must withdraw if the lawyer's physical or mental condition will materially impair his ability to represent the client. [ABA Model Rule 1.16(a)(2)] The client may be right in thinking that hard work will be good for the attorney, but the attorney has to be the ultimate judge of his own physical and mental capacity to carry on. If the attorney believes that his condition prevents him from serving the client competently, he must withdraw regardless of what the client wants. (A) is wrong because the files include confidential information about the client's financial affairs, and the attorney cannot turn them over to his law partner against the client's express wishes. [ABA Model Rule 1.6] (C) is wrong because the client has asked the attorney to complete the work promptly. The attorney's recovery may take months or years. The attorney must not continue representing the client unless he can complete the work with reasonable diligence and promptness. [ABA Model Rule 1.3] (D) is wrong because, as discussed above, if the attorney believes his mental and physical conditions prevent him from serving the client competently, he must withdraw.

A judge sits on a federal appellate court. He and two other federal judges heard a diversity of citizenship case in which they were required to interpret a state statute concerning the marital communications privilege. The judge's two colleagues wrote the majority opinion, in which they concluded that the statute gives only the witness-spouse the right to claim the privilege. The judge wrote a vigorous and scholarly dissent, arguing that the statute gives both spouses the right to claim the privilege. Later, a state senator introduced a bill to amend the statute to reflect the judge's position. The state senate invited the judge to testify about the public policy reasons for giving both spouses the right to claim the privilege. May the judge testify?

Yes, because a judge may engage in activities designed to improve the law. The judge may testify at a public hearing in connection with matters concerning the law. [CJC Rule 3.2(A)] (A) is wrong because there is no rule requiring "equal time." (C) is wrong because it is overbroad. The general rule against judicial involvement in politics limits only some types of political activities, not including legislative testimony. [CJC Canon 4] (D) is wrong because, with respect to issues that are likely to come before the court, a judge is prohibited from making pledges, promises, or commitments that are inconsistent with the impartial performance of his duties. [CJC Rule 4.1(A)(13)] That Rule does not apply here because the judge's testimony, which would be designed to improve the law, would not constitute a promise that is inconsistent with the performance of his adjudicative duties.

A partner in a law firm has represented a local manufacturing company for many years. The company gives the bulk of its legal business to the partner. The partner also does some collection work for the company. A number of the company's customers have not paid their bills, and the partner is in the process of obtaining judgments against them. These particular judgments are all default judgments, as none of the customers have filed answers to the complaints within the time limit stated. Thus, the judgments will be handled in a routine manner by the court with virtually automatic rulings in favor of the company. In the meantime, an associate in the partner's law firm who has never done any work for the company has been retained by a telecommunications company to draw up a number of contracts. The partner files the papers for default judgments against the manufacturing company's delinquent customers. Among these customers is an unincorporated division of the telecommunications company, a fact that is not discovered by the partner until after he has filed the papers. The partner tells the associate who, in turn, tells the appropriate officer of the telecommunications company that he will have to withdraw from representing his company because of a conflict of interest. After explaining the problem fully to the telecommunications company's officer, they part on very cordial terms with the associate being given permission to withdraw. The partner proceeds with his cases against the manufacturing company's delinquent customers. Is the law partner subject to discipline?

Yes, because he did not obtain informed consent, confirmed in writing, from the manufacturing company. Here, there is a potential conflict because the partner's actions against the unincorporated division of the telecommunications company could be affected by the firm's past and possibly future representation of the telecommunications company. It would be proper that disclosure of the conflict of interest be made to the manufacturing company. [ABA Model Rule 1.7(b)]

An attorney represents the plaintiff in a personal injury case. The plaintiff has authorized her attorney to settle the case for $2,000. She further tells him that if he receives $2,000 from the defendant, he may keep $750 of it as his fee, and that he should pay the physician who examined her $250 for his examination and treatment of her injuries. The attorney reaches a $2,000 settlement with the defendant. Upon receipt of a $2,000 check from the defendant, the attorney immediately places $750 in his personal account, sends the plaintiff's physician a check for $250, and places $1,000 in his clients' trust account at the local bank. Sixty days after the attorney received the $2,000 check, his client calls him to inquire about any progress made on a settlement of her case. The attorney tells her that he has settled the case and paid her physician. Immediately after he hangs up the phone, he writes out a check to his client in the amount of $1,000 and mails the check on his lunch hour. Is the attorney subject to discipline?

Yes, because he did not promptly pay his client the money due her. not: Yes, because he paid himself his fee without informing his client. - allowed to pay self money earned

Two lawyers represent two corporations who oppose each other in a civil suit. One of the lawyers has filed a petition with the court, seeking to have the other lawyer removed as opposing counsel. She claims he has a conflict of interest because he once did certain work for her present client. He calls her and tells her that he thinks the conflict of interest matter can be resolved if she listens to his explanation. The attorneys meet and begin to negotiate the conflict of interest issue, and during the same meeting discuss a settlement of the lawsuit. They agree to meet again and continue their discussions. Immediately after the first meeting, the attorney who raised the issue of conflict of interest goes to her office and dictates a letter to the judge who is trying the case, telling the judge that she and the opposing counsel are attempting to resolve their differences on the conflict of interest matter and are also negotiating a settlement of the underlying lawsuit. She mails the letter to the judge that afternoon. Is the attorney subject to discipline?

Yes, because she did not send opposing counsel a copy of the letter. ABA Model Rule 3.5(b) prohibits ex parte communications by a lawyer with a judge except as permitted by law. Lawyers may generally communicate with a judge in writing if the lawyer sends a copy to the adversary. [Restatement §113, comment c] Thus the lawyer's letter could be proper only if a copy was sent to the opposing counsel. Thus, (C) is correct, and (A) and (B) are incorrect. (D) is incorrect; the lawyer need not inform opposing counsel before writing to the judge.

A married couple experienced continuing marital difficulties and, being unable to resolve their problems, decided to divorce. The wife retained an attorney, who filed papers on the husband. Shortly after he was served, the husband telephoned his wife's attorney and suggested that the matter could be resolved amicably. The wife's attorney asked the husband if he was represented by counsel. The husband replied that he was not. The wife's attorney strongly urged the husband to retain a lawyer to represent him and to safeguard his interests, but the husband insisted that he did not need a lawyer. A few weeks later, after ascertaining that the husband still had not retained counsel, the wife's attorney sent the husband a proposed settlement dividing the marital property. There were spaces at the bottom for both the husband and the wife to sign, but the copy sent to the husband was unsigned by the wife. In his cover letter to the husband, the wife's attorney indicated that the husband could sign the settlement agreement, which the attorney characterized as being fair and equitable. However, the attorney's letter also stated that he could not advise the husband as to whether he should sign, and strongly urged the husband to obtain independent counsel to review the papers before signing them. Were the attorney's actions in handling the divorce proper?

Yes, because the attorney did not give the husband legal advice. ABA Model Rule 4.3 forbids an attorney to give advice to an unrepresented person if the lawyer knows that the person's interests conflict with those of the client. The attorney is allowed to advise the unrepresented person to obtain a lawyer, but the critical issue is giving advice.

During the course of researching a reply brief, an attorney discovered a case in the controlling jurisdiction that seemed to be right on point on one of the key issues involved in the case on which she was working. Although much of the dicta in the case seemed to favor the attorney's client, one critical sentence in the holding clearly put the court behind the position asserted by opposing counsel. However, the opposing counsel had done a slipshod job of researching the issues and had failed to find the case and to cite it in his brief. The attorney decided to cite the case in her reply brief, but she cited the case as favoring her client by quoting much of the dicta and deliberately omitting the key sentence in the holding. The attorney filed the reply brief with the court and sent a copy to the opposing counsel, who she knew often bragged about not having read a case since law school. The attorney was also aware that this judge's clerks tended to be overworked and so did not always read all the cases cited by attorneys in their briefs. Therefore, she hoped that the negative aspects of the case might slip by unnoticed by the judge and opposing counsel. Is the attorney subject to discipline?

Yes, because the attorney is attempting to mislead the tribunal. An attorney is subject to discipline for making a false statement of law to the court. [ABA Model Rule 3.3(a)(1)] (A) is overbroad because an attorney need not cite opposing cases from noncontrolling jurisdictions, and an attorney may argue the case from an advocate's viewpoint, which may not necessarily be "objective." (C) and (D) are incorrect because they do not take into account the stricture against making a false statement of law. The attorney here has gone beyond the bounds of zealous representation.

An airline passenger was involved in an aviation mishap. The airline company has admitted liability and has settled with 10 other persons involved in the mishap for amounts ranging between $120,000 and $150,000. The passenger's injuries are very similar to those suffered by the persons with whom the airline has settled. The passenger received a settlement offer of $135,000 from the airline company. Upon receiving the offer, the passenger decided to employ counsel to determine if the offer was a fair amount, and generally to read over the settlement papers and the release that the airline asked the passenger to sign to get the $135,000. The passenger went to the offices of a local attorney, bringing the settlement papers and release with her. She asked the attorney what his hourly fee for reading the papers would be. The attorney told the passenger that she had a personal injury case and that his standard fee for personal injury cases was 30% of any settlement or judgment received by the plaintiff. Is the attorney subject to discipline?

Yes, because the attorney's fee bears no rational relationship to the time and effort required to perform the work requested by the client. The usual rationale supporting relatively high contingent fees is that the lawyer is taking a gamble in handling the case. Here, no gamble is involved, and the attorney is subject to discipline under ABA Model Rule 1.5(a) for attempting to exact an unreasonable fee. The criteria of (A) more clearly establish excessiveness than those in (B). A - Yes, because the attorney's fee bears no rational relationship to the time and effort required to perform the work requested by the client. B - Yes, because 30% is an unreasonable contingent fee percentage.

An attorney had an office in a small town located on the extreme western border of the state in which he was licensed to practice. The attorney received a retainer from a client, with the agreement that the attorney would use funds from the retainer for such things as filing fees when they came due. There was no federally insured bank or savings institution in the small town in which the attorney's office was located. The nearest such institution in the state was located in the county seat, which was over 60 miles away from the attorney's office and where the attorney and the client resided. Therefore, the attorney decided, with the client's consent, to put the client's money in his client trust account in a bank in a medium-sized city located just across the state line in a neighboring state. The account was fully insured by the federal government, but was not an interest-bearing account. The attorney is not licensed to practice law in that state. Was it proper for the attorney to place the client's money in an account in the neighboring state?

Yes, because the client consented to the deposit in the neighboring state. A client's funds are ordinarily deposited in an account where the lawyer's office is located, but they can be deposited elsewhere with the client's consent. [ABA Model Rule 1.15(a)] (A) is wrong because the client's funds appear to be an expense advance, not a "true retainer fee." (C) and (D) are wrong because there are no such requirements. (c) No, because the attorney is not licensed to practice in the neighboring state. (d) No, because the funds were not placed in an interest-bearing account.

A local attorney has recently defended a client in a civil action tried before a jury. The client lost the case, and the plaintiff was awarded a substantial amount of damages. The attorney receives an anonymous, handwritten letter stating that one of the jurors in the plaintiff's case was bribed to influence other members of the jury to side with the plaintiff and to award a large sum in damages. The lawyer hires a local private investigator to investigate the juror to determine if the anonymous charges are true, and instructs the investigator to take care to do nothing that would involve coercion or harassment. Is it proper for the attorney to hire the private investigator?

Yes, because the investigation is to be conducted in such a manner as not to harass the jurors. ABA Model Rule 3.5 prohibits a lawyer from communicating with a discharged juror if the communication involves misrepresentation, coercion, duress, or harassment. [ABA Model Rule 3.5(c)(3)] Thus, the lawyer can hire a private investigator only if the investigation is not conducted in a harassing manner. does not need to notify the juror of his investigation or obtain her consent new evidence will not justify the investigation if it is conducted in an improper manner.

A lawyer represented a defendant in a felony case that charged him with intentional evasion of over $9 million in state income tax. The accused allegedly earned the unreported income from selling child pornography. The state's criminal procedure law requires a unanimous jury verdict to convict in a felony case. Midway through the prosecutor's case-in-chief, the defense lawyer instructed one of the accused's employees to attend the trial daily, to sit in the spectator section as close as possible to the defense counsel table, and to put the "evil eye" on a particular juror to intimidate him. The employee, a wicked-looking man with a jagged scar down the side of his face, attended every court session and stared constantly with cold, squinting eyes at the juror. One day, when no one but that juror was looking, the employee pointed his hand like a pistol directly at the juror's head and pretended to shoot by moving his thumb downward. Then the employee smiled an evil smile and continued to stare. The juror was too frightened to tell anyone, but he endured the torment day after day. Finally, the case was submitted to the jury, and the juror joined the rest of the jurors in returning a prompt, unanimous verdict of guilty. Is the defense lawyer subject to criminal liability for what the employee did?

Yes, because the lawyer induced the employee to intimidate the juror for the purpose of influencing the jury verdict. All jurisdictions in the United States have obstruction of justice statutes that prohibit threatening jurors, witnesses, and judges and also prohibit similar misconduct. For example, 18 U.S.C. section 1503 makes it a felony to "endeavor . . . to influence, intimidate, or impede any . . . juror." The term "endeavor" is broader than "attempt," and one can be guilty of endeavoring to intimidate a juror even though the juror is not intimidated. [See also United States v. Atkin, 107 F.3d 1213 (6th Cir. 1997)-attorney violated section 1503 by obtaining money from defendant to bribe judge, even though attorney ultimately did not offer the bribe]

An amateur inventor comes to an attorney with an invention he wants to patent. The inventor explains that he and a competitor have been racing one another to come up with the ideal cleaning solution. If the inventor's competitor were to find out that the inventor was at the patent stage, and worse, if he found out the inventor's formula, the inventor would be ruined. The attorney, a trained and certified patent attorney, agrees to represent the inventor in the patent process. The invention involves complex chemical formulae, and the attorney's particular area of expertise is electronic devices. However, having worked with inventions of all types, she has no doubt that she can properly shepherd the solution through the patent process. In putting together the necessary paperwork, the attorney asks two associates in her firm who hold chemistry degrees to help her out on the project. In due time, the inventor's product receives a patent. The total bill for legal fees was $60,000, which was reasonable for the work done. When the attorney received the inventor's final payment, she decided to give the two associates each a $10,000 bonus from the fee. Are the attorney's actions proper?

Yes, because with the aid of her associates, she was competent to handle the matter. limitations on fee splitting between lawyers in different firms, not apply to lawyers in the same firm type of disclosure that is impliedly authorized to carry out the representation. It does not abrogate the protections of privilege or confidentiality, and no formal consent by the client is required. [See ABA Model Rule 1.6(a)] No client consent is required to split a fee in any manner among lawyers in the same firm. [ABA Model Rule 1.5]

A Hollywood movie producer was charged under a criminal statute for unfair trade practices, and now faces a civil claim under the same statute. The producer retains an attorney to represent him in both suits. The attorney is a nationally known defense attorney who has represented many famous people. Most recently, he defended a celebrity in a notorious murder case that held the country rapt for several weeks. The attorney explains to the producer that the representation is very complex and would take a majority of his time for several months. Given the attorney's steep hourly rate, the producer's legal fees would likely be around $1 million. The producer is short on cash and makes the following proposal: If the attorney will represent him in both the civil and criminal suits, the producer will produce a movie based on the attorney's most famous past cases, told from the attorney's viewpoint. The attorney would have complete creative control and would be entitled to all of the movie's profits, which could be anything from $0 to $100 million. The producer had his personal attorney draw up a proposal to this effect and submitted it to the attorney. Assuming that the attorney receives any consent necessary from his former clients who might be portrayed in the movie, is this proposed arrangement proper?

Yes, but only if the ultimate amount paid to the attorney is not excessive in light of the work done. An attorney may enter into a business relationship with a client, provided certain safeguards, such as an opportunity to consult with independent counsel, are used. [ABA Model Rule 1.8(a)] (A) is wrong because although it is improper to use a contingent fee in a criminal case, this fee is not a contingent fee. Whether the attorney gets paid does not depend on the outcome of the case; it depends on how well the movie does. (C) is wrong because it is not clear that any amount over $1 million is excessive, given that the attorney is risking that he will be paid nothing and is delaying payment by a substantial period of time. (D) is wrong because it misstates or incompletely states the Rule. The Rule is that prior to the conclusion of the representation of the client, an attorney cannot acquire media rights to a story based substantially on information relating to the representation. In this case, the movie does not relate to the current representation, rather it involves past representations in the attorney's career. While the attorney could not disclose any information related to those representations without the clients' consents, the acquisition of such media rights is not improper.

The state bar association has established a peer counseling program whereby lawyers who are addicted to alcohol or other drugs can receive confidential counseling from other lawyers. The bar association's ethics rule on confidential information provides that communications between the counselor lawyer and the counseled lawyer are to be treated just like confidential communications between an attorney and client. A lawyer is addicted to alcohol and is receiving peer counseling under the program from another lawyer. The lawyer is a large, strong man, and his addiction has made him subject to periodic fits of physical violence. This afternoon, during their peer counseling session, the lawyer told his peer counselor that his client had refused to pay the fees he owes, and that he intended to punch out the client the next time he got roaring drunk. From working with the lawyer over an extended period, the peer counselor believes that he may really do it. May the peer counselor disclose the lawyer's statement to the client and the police?

Yes, even if the lawyer objects. The peer counselor may disclose the statement even if the lawyer objects. The state ethics rule on confidentiality treats communications between a lawyer and his peer counselor just like communications between an attorney and a client. If the counselor had heard one of her clients make this threat, she could have warned the police and the intended victim. An attorney may reveal confidential information to the extent she reasonably believes necessary to prevent reasonably certain death or substantial bodily harm. [ABA Model Rule 1.6(b)(1)] Thus, the peer counselor may warn the client and the police. (A) is wrong because the peer counselor may act to prevent the lawyer from causing substantial bodily harm. The lawyer's consent is not necessary. [ABA Model Rule 1.6(b)(1)] (B) is wrong because if an attorney reasonably believes that her client (or anyone else) is about to inflict substantial bodily harm on someone, she may take steps to prevent it, even if she is not certain that the client (or other person) will do it. (D) is wrong because the state ethics rule on confidentiality equates the peer counselor relationship with the relationship between an attorney and client; thus, the ability to disclose is the same.

A swimming coach was charged with assault of another coach. The swimming coach hired a criminal attorney to defend him. Subsequently, the swimming coach pleaded not guilty and was released on his own recognizance. At his first trial, a jury was empanelled, and the prosecutor was almost finished presenting the testimony of her first witness when a signal from her electronic pager interrupted her. The trial judge granted her request for a short recess, at the end of which the prosecutor told the judge that her office had instructed her not to proceed with this case at this time. The judge responded that if the prosecutor stopped now, the defendant would go free. When the prosecutor indicated that she understood, the judge entered a judgment of acquittal and set the swimming coach free. Twenty days later, the prosecutor recharged the swimming coach with the same offense. The swimming coach hired his original criminal attorney to defend him. The same judge presided over the second trial. The swimming coach's attorney made no pretrial motions. This time the prosecutor did not falter, and in due course the jury at the second trial found the swimming coach guilty as charged. The judge sentenced him to prison for the period required by law, but she stayed the sentence and released him on his own recognizance pending appeal. The swimming coach reluctantly paid the criminal attorney's bill for the second trial-$5,000. However, the swimming coach hired a new lawyer for the appeal, and in due course the appellate court reversed the conviction and set aside the prison sentence. The appellate court's opinion stated it had never seen a clearer double jeopardy violation. Will the swimming coach's original criminal attorney be subject to civil liability in a legal malpractice action brought by the swimming coach for having missed the double jeopardy issue?

Yes, provided that the swimming coach proves by a preponderance of evidence that he did not commit the assault on the opposing coach. The criminal attorney will be subject to civil liability in a legal malpractice action brought by the swimming coach for failing to object to the second trial on double jeopardy grounds. A reasonably competent criminal defense attorney would know that a defendant is put in jeopardy when a jury is empanelled and sworn, not to mention that the prosecutor started presenting her case-in-chief. The swimming coach was obviously a proper plaintiff, and he was injured by the criminal attorney's error; he should be able to recover at least part of the $5,000 attorneys' fee, plus damages for his anguish and for the reputational injury caused by the conviction at the second trial. Note that (C) provides that in the malpractice action the swimming coach must prove by a preponderance of the evidence that he was innocent of the underlying criminal offense. That is required by the law of most states that have ruled on the issue. [See Restatement §53, comment d] Observe that in this particular case, a good argument can be made for allowing the swimming coach to recover even without proof of innocence. Here, the malpractice was the criminal attorney's failure to object to the second trial. If the attorney had acted competently, the second trial would never have taken place, and the swimming coach would have lawfully gone free, even if he were unquestionably guilty of the assault. [See Levine v. Kling, 123 F.3d 580 (7th Cir. 1997)-Judge Posner's dictum] (A) and (D) are incorrect because they overlook the $5,000 fee and other less tangible injuries the swimming coach suffered. (B) is incorrect for the reasons stated above with respect to (C).

A client hired a lawyer to draft a will for him. The client willed his entire estate to a 43-year-old widow. The client told the lawyer in confidence that he was neither a relative nor a friend of the widow. The client explained that he felt a moral obligation to the widow because he had killed her husband, and he had never become a suspect or confessed his sin to anyone. One day after signing the will, the client committed suicide. In due course, all of the client's assets were distributed to the widow, and the probate court closed his estate and discharged his executor. The lawyer never told the widow or anyone else that the client had confessed to killing the widow's husband. Now, a few years later, an enthusiastic young prosecutor is charging an innocent man with murdering the widow's husband in the first degree with aggravating circumstances, and the prosecutor is seeking the death penalty. May the lawyer voluntarily tell the innocent man's defense counsel what his client told him in confidence about killing the widow's husband?

Yes, the lawyer may tell, but he would not be subject to discipline if he decides not to do so. The controlling doctrine in this case is the lawyer's ethical duty of confidentiality, not the attorney-client privilege. The lawyer needs to know whether he can voluntarily reveal the client's confession, not whether he would be forced to do so if he were put on the witness stand in a court. ABA Model Rule 1.6(b)(1) states the applicable exception to the ethical duty of confidentiality: A lawyer may reveal confidential information if the lawyer reasonably believes that doing so is necessary to prevent reasonably certain death or substantial bodily harm. One might quibble whether the innocent man's death is "reasonably certain" when his trial has not even started, but surely the ethics rule should not be read to require the innocent man to order his last meal before being loosed from the executioner's grip. (A) is wrong because ABA Model Rule 1.6(b)(1) gives the lawyer discretion to reveal the client's confession; the Rule does not force him to do so. [See comment 15 to ABA Model Rule 1.6] (A few states go farther and require disclosure to prevent death or substantial bodily harm, but they are a small minority.) (C) is wrong for two reasons. First, the applicable doctrine is the ethical duty of confidentiality, not the attorney-client privilege. Second, even if the privilege were the applicable doctrine, who could claim it in this situation? The client cannot because he is dead. The client's executor cannot because the client's estate was closed and the executor was discharged. The lawyer cannot claim it because a lawyer's right to claim the privilege is only derivative from the client. (D) is wrong for two reasons. First, the admissibility of this hearsay is irrelevant to the ethics issue. Second, the client's confession would likely be admissible if offered by the innocent man against the prosecution because it is a declaration against penal interest by an unavailable declarant, and the client's will and suicide are independent evidence of the confession's trustworthiness. [See Fed. R. Evid. 804(b)(3); see also Chambers v. Mississippi, 410 U.S. 284 (1973)-due process violation where another man's confession was excluded in a murder trial]


Kaugnay na mga set ng pag-aaral

La Emigración | Articulo en español

View Set

exam 2: (ch 45) the child with a respiratory alteration

View Set

Test 3- Elimination and Inflammation

View Set

Canadian Immigration and Refugee Law Chapter 5

View Set

Ultimate AP Gov Study Guide IWA Ch.1-3

View Set

Med Surg Test 3 NCLEX select all that apply

View Set